102
r t LSAT PrepTest 1 www.review.com 800-2-REVIEW www.review.com 800-2-REVIEW Copyright © 1999 by Princeton Review Management, L.L.C. All Rights Reserved. Section 1 1 2 3 4 5 6 7 8 9 10 11 12 13 14 15 16 17 18 19 20 21 22 23 24 25 26 27 Section 4 1 2 3 4 5 6 7 8 9 10 11 12 13 14 15 16 17 18 19 20 21 22 23 24 25 26 27 Section 3 1 2 3 4 5 6 7 8 9 10 11 12 13 14 15 16 17 18 19 20 21 22 23 24 25 26 27 Section 2 1 2 3 4 5 6 7 8 9 10 11 12 13 14 15 16 17 18 19 20 21 22 23 24 25 26 27 Section 1 RC 1 2 3 4 5 6 7 8 9 10 11 12 13 14 15 16 17 18 19 20 21 22 23 24 25 26 27 28 Section 2 Games 1 2 3 4 5 6 7 8 9 10 11 12 13 14 15 16 17 18 19 20 21 22 23 24 Section 3 Args 1 1 2 3 4 5 6 7 8 9 10 11 12 13 14 15 16 17 18 19 20 21 22 23 24 25 Section 4 Args 2 1 2 3 4 5 6 7 8 9 10 11 12 13 14 15 16 17 18 19 20 21 22 23 24 index

rt PrepTest 1 LSAT index - The Princeton Review · PDF filert PrepTest 1 LSAT 800-2-REVIEW ... examined in the second paragraph, but the author’s main idea is to focus on and evaluate

  • Upload
    vocong

  • View
    225

  • Download
    0

Embed Size (px)

Citation preview

rt LSATPrepTest 1

www.review.com 800-2-REVIEWwww.review.com 800-2-REVIEWCopyright © 1999 by Princeton Review Management, L.L.C. All Rights Reserved.

Section 1

1

2

3

4

5

6

7

8

9

10

11

12

13

14

15

16

17

18

19

20

21

22

23

24

25

26

27

Section 4

1

2

3

4

5

6

7

8

9

10

11

12

13

14

15

16

17

18

19

20

21

22

23

24

25

26

27

Section 3

1

2

3

4

5

6

7

8

9

10

11

12

13

14

15

16

17

18

19

20

21

22

23

24

25

26

27

Section 2

1

2

3

4

5

6

7

8

9

10

11

12

13

14

15

16

17

18

19

20

21

22

23

24

25

26

27

Section 1RC1

2

3

4

5

6

7

8

9

10

11

12

13

14

15

16

17

18

19

20

21

22

23

24

25

26

27

28

Section 2Games

1

2

3

4

5

6

7

8

9

10

11

12

13

14

15

16

17

18

19

20

21

22

23

24

Section 3Args 1

1

2

3

4

5

6

7

8

9

10

11

12

13

14

15

16

17

18

19

20

21

22

23

24

25

Section 4Args 2

1

2

3

4

5

6

7

8

9

10

11

12

13

14

15

16

17

18

19

20

21

22

23

24

index

rt LSATPrepTest 1

www.review.com 800-2-REVIEWwww.review.com 800-2-REVIEWCopyright © 1999 by Princeton Review Management, L.L.C. All Rights Reserved.

section 1question 1

Passage OneSynopsis: Phillis Wheatley was brought over from Africa to colonial New England as a slave. She learnedEnglish and wrote poetry in the style of eighteenth-century English poetry. Although the author admiresher accomplishments, the author feels that Wheatley could have done more to advance an AfricanAmerican literary language had she incorporated more of her native culture into her writings. The differ-ences between African oral tradition and the conventions of eighteenth-century English poetry areexamined in the second paragraph, but the author’s main idea is to focus on and evaluate Wheatley’saccomplishments.

Question 1: What is the main idea? Your answer should mention Phillis Wheatley in a generally positiveway but should account for the fact that the author feels Wheatley could have done more to contribute toa distinctive African American literary language.

(A) Although the focus of the passage is on Wheatley and her poetry, the author compares her poetry toits potential. The author’s primary focus is not to compare Wheatley’s poetry to the work of folkartists.

(B) This answer captures about half of what the author did. The author also evaluates Wheatley’s poetryagainst its potential as a first step in developing a distinctive African American literary language.

(C) This is the credited response. See the above synopsis for more detail.(D) This contradicts information in the passage. The author claims that Phillis Wheatley did not contrib-

ute to a distinctive African American literary language.(E) No mention is made of Phillis Wheatley doing anything jointly with African American folk artists.

Although the author claims that Wheatley’s experience exemplifies the meeting of oral and writtenliterary cultures (African literary culture was an oral culture while Western literary culture was a writtenone), it is clear in the passage that Wheatley conformed to the standards of eighteenth-centuryEnglish poetry at the expense of remaining true to the aesthetic conventions of the African Americanoral literary culture.

rt LSATPrepTest 1

www.review.com 800-2-REVIEWwww.review.com 800-2-REVIEWCopyright © 1999 by Princeton Review Management, L.L.C. All Rights Reserved.

section 1question 2

Passage OneSynopsis: Phillis Wheatley was brought over from Africa to colonial New England as a slave. She learnedEnglish and wrote poetry in the style of eighteenth-century English poetry. Although the author admiresher accomplishments, the author feels that Wheatley could have done more to advance an AfricanAmerican literary language had she incorporated more of her native culture into her writings. The differ-ences between African oral tradition and the conventions of eighteenth-century English poetry areexamined in the second paragraph, but the author’s main idea is to focus on and evaluate Wheatley’saccomplishments.

Question 2: Analogy. If a modern-day Italian immigrant in America were to approach poetry in a manneranalogous to Phillis Wheatley’s approach, what would be the result? The passage indicates that PhillisWheatley adopted the literary forms of eighteenth-century English poetry at the expense of remaining trueto the aesthetic conventions of her native culture. An analogous approach would see the Italian immigrantadopting the forms of American poetry.

(A) There is no mention of Phillis Wheatley doing any translating of African literary forms.(B) Wheatley did not invent a new form of poetic expression. She merely adopted the conventions of

eighteenth-century English poetry.(C) This answer is very much like answer choice (B) and is wrong for the same reasons.(D) Wheatley defined artistic expression in the terms of her adopted culture. This answer has the Italian

poet defining artistic expression in the terms of her native culture.(E) This is the credited response. See the above explanation for more detail.

rt LSATPrepTest 1

www.review.com 800-2-REVIEWwww.review.com 800-2-REVIEWCopyright © 1999 by Princeton Review Management, L.L.C. All Rights Reserved.

section 1question 3

Passage OneSynopsis: Phillis Wheatley was brought over from Africa to colonial New England as a slave. She learnedEnglish and wrote poetry in the style of eighteenth-century English poetry. Although the author admiresher accomplishments, the author feels that Wheatley could have done more to advance an AfricanAmerican literary language had she incorporated more of her native culture into her writings. The differ-ences between African oral tradition and the conventions of eighteenth-century English poetry areexamined in the second paragraph, but the author’s main idea is to focus on and evaluate Wheatley’saccomplishments.

Question 3: This question should inspire you to go back to the passage for the answer. The discussion ofthe influence of African languages takes place in the middle of the second paragraph. On lines 19-24, thereis direct evidence that African languages had a notable influence on American speech patterns (answerchoice (D)). “(African languages) exerted demonstrable effects on the manner in which other Americansspoke English.”

rt LSATPrepTest 1

www.review.com 800-2-REVIEWwww.review.com 800-2-REVIEWCopyright © 1999 by Princeton Review Management, L.L.C. All Rights Reserved.

section 1question 4

Passage OneSynopsis: Phillis Wheatley was brought over from Africa to colonial New England as a slave. She learnedEnglish and wrote poetry in the style of eighteenth-century English poetry. Although the author admiresher accomplishments, the author feels that Wheatley could have done more to advance an AfricanAmerican literary language had she incorporated more of her native culture into her writings. The differ-ences between African oral tradition and the conventions of eighteenth-century English poetry areexamined in the second paragraph, but the author’s main idea is to focus on and evaluate Wheatley’saccomplishments.

Question 4: On a question containing a line reference, it’s a good idea to put that line in context. In thisinstance, start at the beginning of the third paragraph and continue about five lines beyond the words“closed system”. This reading should reveal that a “closed system” of poetry relies on imitation ofprevious works and does not allow for use of the “American language”. From lines 36-40: “No placeexisted for the rough-and-ready Americanized English she heard in the streets . . . The conventions ofeighteenth-century neoclassical poetry ruled out casual talk”. This idea is best captured in answer choice(D).

rt LSATPrepTest 1

www.review.com 800-2-REVIEWwww.review.com 800-2-REVIEWCopyright © 1999 by Princeton Review Management, L.L.C. All Rights Reserved.

section 1question 5

Passage OneSynopsis: Phillis Wheatley was brought over from Africa to colonial New England as a slave. She learnedEnglish and wrote poetry in the style of eighteenth-century English poetry. Although the author admiresher accomplishments, the author feels that Wheatley could have done more to advance an AfricanAmerican literary language had she incorporated more of her native culture into her writings. The differ-ences between African oral tradition and the conventions of eighteenth-century English poetry areexamined in the second paragraph, but the author’s main idea is to focus on and evaluate Wheatley’saccomplishments.

Question 5: This question will most likely be answered with information found in the general neighbor-hood of the information you read to answer question 4. From lines 40-41: “her (Wheatley’s) voice andfeelings had to be generalized according to rules of poetic diction and characterization”. These linessupport answer choice (A).

rt LSATPrepTest 1

www.review.com 800-2-REVIEWwww.review.com 800-2-REVIEWCopyright © 1999 by Princeton Review Management, L.L.C. All Rights Reserved.

section 1question 6

Passage OneSynopsis: Phillis Wheatley was brought over from Africa to colonial New England as a slave. She learnedEnglish and wrote poetry in the style of eighteenth-century English poetry. Although the author admiresher accomplishments, the author feels that Wheatley could have done more to advance an AfricanAmerican literary language had she incorporated more of her native culture into her writings. The differ-ences between African oral tradition and the conventions of eighteenth-century English poetry areexamined in the second paragraph, but the author’s main idea is to focus on and evaluate Wheatley’saccomplishments.

Question 6: Weaken the author’s argument concerning Wheatley’s contribution to the evolution of anAfrican American literary language. The author claims that “Wheatley’s poetry contributed little to thedevelopment of a distinctive African American literary language.” (lines 54-55). Consequently, you arelooking for an answer that claims that Wheatley made a significant contribution to this end. Although (D)is a close second choice, a closer reading of that answer choice should reveal that the answer choice refersto the overbroad “American literature” rather than “African American literature” (end of answer choice).Instead, the credited response is (E). This answer affirms that Wheatley did make a contribution to theevolution of an African American literary language, thereby weakening the author’s position.

rt LSATPrepTest 1

www.review.com 800-2-REVIEWwww.review.com 800-2-REVIEWCopyright © 1999 by Princeton Review Management, L.L.C. All Rights Reserved.

section 1question 7

Passage OneSynopsis: Phillis Wheatley was brought over from Africa to colonial New England as a slave. She learnedEnglish and wrote poetry in the style of eighteenth-century English poetry. Although the author admiresher accomplishments, the author feels that Wheatley could have done more to advance an AfricanAmerican literary language had she incorporated more of her native culture into her writings. The differ-ences between African oral tradition and the conventions of eighteenth-century English poetry areexamined in the second paragraph, but the author’s main idea is to focus on and evaluate Wheatley’saccomplishments.

Question 7: According to the author, what is the shortcoming of Wheatley’s poetry? By referring to thepassage synopsis on the previous page, you should recall that the author feels that although Wheatleywas an accomplished poet, she could have accomplished more had she incorporated elements of herAfrican culture into her poetry. This idea is best captured in answer choice (D).

rt LSATPrepTest 1

www.review.com 800-2-REVIEWwww.review.com 800-2-REVIEWCopyright © 1999 by Princeton Review Management, L.L.C. All Rights Reserved.

section 1question 8

Passage OneSynopsis: Phillis Wheatley was brought over from Africa to colonial New England as a slave. She learnedEnglish and wrote poetry in the style of eighteenth-century English poetry. Although the author admiresher accomplishments, the author feels that Wheatley could have done more to advance an AfricanAmerican literary language had she incorporated more of her native culture into her writings. The differ-ences between African oral tradition and the conventions of eighteenth-century English poetry areexamined in the second paragraph, but the author’s main idea is to focus on and evaluate Wheatley’saccomplishments.

Question 8: What is the author’s attitude towards Wheatley’s literary accomplishments? The answer tothe previous two questions should act as a guide. The author is generally positive about Wheatley, but isnot entirely so (because Wheatley could have done more to advance an African American literary lan-guage). This idea is best captured in answer choice (B).

rt LSATPrepTest 1

www.review.com 800-2-REVIEWwww.review.com 800-2-REVIEWCopyright © 1999 by Princeton Review Management, L.L.C. All Rights Reserved.

section 1question 9

Passage TwoSynopsis: A general idea (the interplay between a scientific discipline and its antithesis) is illustrated witha specific example (cell biology versus biochemistry). The author takes a neutral stance and is simplyobserving the phenomenon that the interplay between a scientific discipline and its antithesis can yieldnew discoveries and disciplines in science (in the specific instance noted the interplay between cellbiology and biochemistry gave rise to the discipline of molecular genetics).

Question 9: Identify the main idea. The author is making a general observation and illustrating it with aspecific example. The answer to the main idea question should focus on the general rather than thespecific example.

(A) There is no mention of antithetical scientific disciplines hindering one another’s research.(B) This is the credited response. See the above analysis.(C) To say that these interactions necessarily lead to a synthesis of their approaches is too strong a

statement given the information presented in the passage.(D) This answer is too specific. The story of cytology versus biochemistry is used to illustrate the

author’s main point which is more general.(E) See (D).

rt LSATPrepTest 1

www.review.com 800-2-REVIEWwww.review.com 800-2-REVIEWCopyright © 1999 by Princeton Review Management, L.L.C. All Rights Reserved.

section 1question 10

Passage TwoSynopsis: A general idea (the interplay between a scientific discipline and its antithesis) is illustrated witha specific example (cell biology versus biochemistry). The author takes a neutral stance and is simplyobserving the phenomenon that the interplay between a scientific discipline and its antithesis can yieldnew discoveries and disciplines in science (in the specific instance noted the interplay between cellbiology and biochemistry gave rise to the discipline of molecular genetics).

Question 10: Which of the following discoveries can be attributed to cytologists in the late nineteenthcentury? This answer will likely be found in the second paragraph. From lines 19-21: “They (cytologists)also deduced the mysterious choreography of the chromosomes during cell division.” This concept isbest paraphrased in answer choice (E).

rt LSATPrepTest 1

www.review.com 800-2-REVIEWwww.review.com 800-2-REVIEWCopyright © 1999 by Princeton Review Management, L.L.C. All Rights Reserved.

section 1question 11

Passage TwoSynopsis: A general idea (the interplay between a scientific discipline and its antithesis) is illustrated witha specific example (cell biology versus biochemistry). The author takes a neutral stance and is simplyobserving the phenomenon that the interplay between a scientific discipline and its antithesis can yieldnew discoveries and disciplines in science (in the specific instance noted the interplay between cellbiology and biochemistry gave rise to the discipline of molecular genetics).

Question 11: Which group (or groups) would most likely participate in the nineteenth-century debate overthe structural nature of protoplasm? The timing of this debate predates the synthesis of the disciplines ofcytology and biochemistry, so you can eliminate choices (D) and (E) right away. The remaining answerchoices can be addressed by referring to the second paragraph. In that paragraph the author distinguishescytology from biochemistry by explaining that cytologists were interested in structures and that biochem-ists were interested in chemical processes. Thus, a debate over the structural nature of protoplasm wouldmost likely be held among cytologists. The credited response is therefore (A).

rt LSATPrepTest 1

www.review.com 800-2-REVIEWwww.review.com 800-2-REVIEWCopyright © 1999 by Princeton Review Management, L.L.C. All Rights Reserved.

section 1question 12

Passage TwoSynopsis: A general idea (the interplay between a scientific discipline and its antithesis) is illustrated witha specific example (cell biology versus biochemistry). The author takes a neutral stance and is simplyobserving the phenomenon that the interplay between a scientific discipline and its antithesis can yieldnew discoveries and disciplines in science (in the specific instance noted the interplay between cellbiology and biochemistry gave rise to the discipline of molecular genetics).

Question 12: According to cytologists, what was the shortcoming of the cell research of biochemists inthe nineteenth century? From lines 35-37: “cytologists considered the methods of biochemists inadequateto characterize the structures of the living cell.” This idea is best captured in answer choice (B).

rt LSATPrepTest 1

www.review.com 800-2-REVIEWwww.review.com 800-2-REVIEWCopyright © 1999 by Princeton Review Management, L.L.C. All Rights Reserved.

section 1question 13

Passage TwoSynopsis: A general idea (the interplay between a scientific discipline and its antithesis) is illustrated witha specific example (cell biology versus biochemistry). The author takes a neutral stance and is simplyobserving the phenomenon that the interplay between a scientific discipline and its antithesis can yieldnew discoveries and disciplines in science (in the specific instance noted the interplay between cellbiology and biochemistry gave rise to the discipline of molecular genetics).

Question 13: Why does the author quote Fruton at the end of the passage? This quote summarizes theauthor’s main idea, consequently the credited response is (A).

rt LSATPrepTest 1

www.review.com 800-2-REVIEWwww.review.com 800-2-REVIEWCopyright © 1999 by Princeton Review Management, L.L.C. All Rights Reserved.

section 1question 14

Passage TwoSynopsis: A general idea (the interplay between a scientific discipline and its antithesis) is illustrated witha specific example (cell biology versus biochemistry). The author takes a neutral stance and is simplyobserving the phenomenon that the interplay between a scientific discipline and its antithesis can yieldnew discoveries and disciplines in science (in the specific instance noted the interplay between cellbiology and biochemistry gave rise to the discipline of molecular genetics).

Question 14: Which of the following answer choices about the enzyme theory of life can be justified fromthe passage?

(A) This is the credited response. The enzyme theory of life was a “newly formulated theory” (lines 31-32) in the late nineteenth century. Molecular biology was a new discipline after 1950 (see the fourthparagraph). Therefore, the enzyme theory was formulated before the appearance of molecular biology.

(B) There is no evidence in the passage to allow you to say for sure whether this is true or not.(C) No information is given with respect to when chromosome mapping had been completed, so this

answer cannot be correct.(D) This contradicts information in the passage.(E) Although the debate over the chemical nature of protoplasm seems to have occurred about the same

time the enzyme theory of life was formulated, there could have been earlier debates over the nature ofprotoplasm in general. Because this question is not answered, you shouldn’t pick this answer choice.

rt LSATPrepTest 1

www.review.com 800-2-REVIEWwww.review.com 800-2-REVIEWCopyright © 1999 by Princeton Review Management, L.L.C. All Rights Reserved.

section 1question 15

Passage TwoSynopsis: A general idea (the interplay between a scientific discipline and its antithesis) is illustrated witha specific example (cell biology versus biochemistry). The author takes a neutral stance and is simplyobserving the phenomenon that the interplay between a scientific discipline and its antithesis can yieldnew discoveries and disciplines in science (in the specific instance noted the interplay between cellbiology and biochemistry gave rise to the discipline of molecular genetics).

Question 15: Find a statement about cells that would have been made by nineteenth-century biochemists.Biochemists of that time felt that the secret to cell structure would be revealed by an understanding of itschemical properties.

(A) This is what cytologists would say.(B) This is the credited response.(C) No mention is made of behavioral patterns of organisms.(D) Biochemists are not interested in physical characteristics. Instead they focus on chemical properties.(E) At no point in the passage does it say that biochemists felt that chemistry alone would be insufficient

to make a full accounting of a cell’s structures.

rt LSATPrepTest 1

www.review.com 800-2-REVIEWwww.review.com 800-2-REVIEWCopyright © 1999 by Princeton Review Management, L.L.C. All Rights Reserved.

section 1question 16

Passage TwoSynopsis: A general idea (the interplay between a scientific discipline and its antithesis) is illustrated witha specific example (cell biology versus biochemistry). The author takes a neutral stance and is simplyobserving the phenomenon that the interplay between a scientific discipline and its antithesis can yieldnew discoveries and disciplines in science (in the specific instance noted the interplay between cellbiology and biochemistry gave rise to the discipline of molecular genetics).

Question 16: Describe the organization of the passage. A general phenomenon is presented and illus-trated through the use of a specific example. The best paraphrase of this idea can be found in answerchoice (C).

rt LSATPrepTest 1

www.review.com 800-2-REVIEWwww.review.com 800-2-REVIEWCopyright © 1999 by Princeton Review Management, L.L.C. All Rights Reserved.

section 1question 17

Passage ThreeSynopsis: An examination of the two major systems of criminal procedure in the modern world: adversarialand inquisitorial. The former system involves both parties in the dispute doing their own fact finding andpresenting their own cases in a procedure viewed as “a forensic duel” (line 19). The latter system involvesboth sides fact finding but there is full disclosure of the facts discovered in advance of the trial. The focusof the inquisitorial system is a search for the truth of the matter. Although there is some discussion ofwhich system may be more favorable to a guilty client, the author remains neutral and does not express apersonal preference of one system over the other.

Question 17: What is the crucial factor in the adversarial system of trials? As is stated on lines 45-48:“This system (inquisitorial) operates on the philosophical premise that in a criminal action the crucial factoris the body of facts, not the legal rule (in contrast to the adversarial system) . . .” This quote can be usedas evidence to defend answer choice (A).

rt LSATPrepTest 1

www.review.com 800-2-REVIEWwww.review.com 800-2-REVIEWCopyright © 1999 by Princeton Review Management, L.L.C. All Rights Reserved.

section 1question 18

Passage ThreeSynopsis: An examination of the two major systems of criminal procedure in the modern world: adversarialand inquisitorial. The former system involves both parties in the dispute doing their own fact finding andpresenting their own cases in a procedure viewed as “a forensic duel” (line 19). The latter system involvesboth sides fact finding but there is full disclosure of the facts discovered in advance of the trial. The focusof the inquisitorial system is a search for the truth of the matter. Although there is some discussion ofwhich system may be more favorable to a guilty client, the author remains neutral and does not express apersonal preference of one system over the other.

Question 18: What is the primary role of a judge in a trial under the inquisitorial system? From lines 41-43:“the inquisitorial system mandates that the judge take an active part in the conduct of a trial, with a rolethat is both directive and protective.” Answer choices (A) and (C) can be eliminated because the judgetakes and active role in the conduct of a trial. Nothing in the passage suggests that a judge would bebiased, and on the LSAT it seems unlikely that something so uncomplimentary would be said aboutjudges, so (B) can be eliminated. The language in answer choice (D) is too strong to describe what thejudge does, but the language in answer choice (E) is a good paraphrase for “a role that is both directiveand protective”.

rt LSATPrepTest 1

www.review.com 800-2-REVIEWwww.review.com 800-2-REVIEWCopyright © 1999 by Princeton Review Management, L.L.C. All Rights Reserved.

section 1question 19

Passage ThreeSynopsis: An examination of the two major systems of criminal procedure in the modern world: adversarialand inquisitorial. The former system involves both parties in the dispute doing their own fact finding andpresenting their own cases in a procedure viewed as “a forensic duel” (line 19). The latter system involvesboth sides fact finding but there is full disclosure of the facts discovered in advance of the trial. The focusof the inquisitorial system is a search for the truth of the matter. Although there is some discussion ofwhich system may be more favorable to a guilty client, the author remains neutral and does not express apersonal preference of one system over the other.

Question 19: When the private vengeance system was abandoned in favor of the adversarial system (andlater the inquisitorial system), “the right to initiate legal action against a criminal (was) extended to allmembers of society” (lines 11-13). That right was formerly held only by the victim of a crime. This idea isbest captured by answer choice (B).

rt LSATPrepTest 1

www.review.com 800-2-REVIEWwww.review.com 800-2-REVIEWCopyright © 1999 by Princeton Review Management, L.L.C. All Rights Reserved.

section 1question 20

Passage ThreeSynopsis: An examination of the two major systems of criminal procedure in the modern world: adversarialand inquisitorial. The former system involves both parties in the dispute doing their own fact finding andpresenting their own cases in a procedure viewed as “a forensic duel” (line 19). The latter system involvesboth sides fact finding but there is full disclosure of the facts discovered in advance of the trial. The focusof the inquisitorial system is a search for the truth of the matter. Although there is some discussion ofwhich system may be more favorable to a guilty client, the author remains neutral and does not express apersonal preference of one system over the other.

Question 20: Which answer choice is not a feature of the inquisitorial system?

(A) This is a feature. Both sides share the information they acquire while investigating the facts of thecase.

(B) This is a feature. See lines 33-38.(C) This is a feature. See lines 41-42.(D) This is the credited response. The prosecutor is also bound to reveal exculpatory evidence, and the

process itself is much more interested in the pursuit of the facts.(E) This is a feature. See lines 53-55.

rt LSATPrepTest 1

www.review.com 800-2-REVIEWwww.review.com 800-2-REVIEWCopyright © 1999 by Princeton Review Management, L.L.C. All Rights Reserved.

section 1question 21

Passage ThreeSynopsis: An examination of the two major systems of criminal procedure in the modern world: adversarialand inquisitorial. The former system involves both parties in the dispute doing their own fact finding andpresenting their own cases in a procedure viewed as “a forensic duel” (line 19). The latter system involvesboth sides fact finding but there is full disclosure of the facts discovered in advance of the trial. The focusof the inquisitorial system is a search for the truth of the matter. Although there is some discussion ofwhich system may be more favorable to a guilty client, the author remains neutral and does not express apersonal preference of one system over the other.

Question 21: Describe the author’s attitude towards the inquisitorial system. The author claims that aninnocent defendant would prefer the inquisitorial system because its focus is on fact finding. Presumablythis is true because the author is confident that the elements that make up the inquisitorial system functionas described.

(A) This would cast doubt on the effectiveness of the inquisitorial system and therefore does not repre-sent the author’s opinion.

(B) This is the credited response.(C) The author doesn’t go quite this far. The author does believe that the inquisitorial system is more

favorable to an innocent defendant, but no claim is made as to whether or not this system will entirelysupplant the adversarial system.

(D) The author does not appear to be wary about any aspect of the inquisitorial system.(E) The author is certainly not critical of any aspect of the inquisitorial system.

rt LSATPrepTest 1

www.review.com 800-2-REVIEWwww.review.com 800-2-REVIEWCopyright © 1999 by Princeton Review Management, L.L.C. All Rights Reserved.

section 1question 22

Passage FourSynopsis: The author defends the practice of medicine as a profession rather than a trade. The authorattempts to accomplish this by examining what does and does not constitute what a “profession”: first byexamining and dismissing some perspectives and finally by settling on his own concept of the idea (apublic pronouncement of a commitment to a way of life that serves a higher good).

Question 22: Which one of the following is required in order to be a professional? According to thesynopsis, one must make a public pronouncement of dedication to a way of life that serves a higher good.This is best captured by answer choice (E).

rt LSATPrepTest 1

www.review.com 800-2-REVIEWwww.review.com 800-2-REVIEWCopyright © 1999 by Princeton Review Management, L.L.C. All Rights Reserved.

section 1question 23

Passage FourSynopsis: The author defends the practice of medicine as a profession rather than a trade. The authorattempts to accomplish this by examining what does and does not constitute what a “profession”: first byexamining and dismissing some perspectives and finally by settling on his own concept of the idea (apublic pronouncement of a commitment to a way of life that serves a higher good).

Question 23: What is the main point of the passage? Your answer should include a mention of thedefense of medicine as a profession and should include the author’s definition of what constitutes aprofession.

(A) The author feels that medicine is defined as a profession because of the public commitment of thepractitioner, not because of the etymology of the word “profession”.

(B) The author would disagree with this point.(C) This answer choice ignores the last eighty percent of the passage.(D) This is the credited response. See the above synopsis for more detail.(E) The passage focuses on why physicians should be considered professionals. It does not address

physicians changing their orientations.

rt LSATPrepTest 1

www.review.com 800-2-REVIEWwww.review.com 800-2-REVIEWCopyright © 1999 by Princeton Review Management, L.L.C. All Rights Reserved.

section 1question 24

Passage FourSynopsis: The author defends the practice of medicine as a profession rather than a trade. The authorattempts to accomplish this by examining what does and does not constitute what a “profession”: first byexamining and dismissing some perspectives and finally by settling on his own concept of the idea (apublic pronouncement of a commitment to a way of life that serves a higher good).

Question 24: Why does the author pose the question of why physicians should resist efforts to define thepractice of medicine as a trade? The question is used as a launching point for the author’s discussion ofwhat is a profession and why medicine should be considered a profession.

(A) The author makes no claim that the effort to continue to define the physician’s work as a profession isa futile one.

(B) These issues are no longer discussed after the question is raised.(C) This is the credited response. The “nature of the practice of medicine” refers to the question of why

it’s not appropriate to think of medicine as a trade. Much of that consideration is rooted in theauthor’s analysis of what constitutes a “profession”.

(D) While the passage is a defense of the consideration of physicians as “professionals”, it can’t beconclusively stated that the passage is intended to rally sensible people. It certainly doesn’t suggesthow that might be done.

(E) Although the author segues from the question cited to a discussion of etymology, the question isposed to set up a broader examination.

rt LSATPrepTest 1

www.review.com 800-2-REVIEWwww.review.com 800-2-REVIEWCopyright © 1999 by Princeton Review Management, L.L.C. All Rights Reserved.

section 1question 25

Passage FourSynopsis: The author defends the practice of medicine as a profession rather than a trade. The authorattempts to accomplish this by examining what does and does not constitute what a “profession”: first byexamining and dismissing some perspectives and finally by settling on his own concept of the idea (apublic pronouncement of a commitment to a way of life that serves a higher good).

Question 25: Questions such as this one require a lot of reading: four of the five answer choices can befound in the passage. The one that can’t be found is the credited response.

(A) This is addressed in the first paragraph.(B) This is done on lines 50-55.(C) This is done on lines 58-60.(D) This is the credited response.(E) This is one of the primary purposes of the passage.

rt LSATPrepTest 1

www.review.com 800-2-REVIEWwww.review.com 800-2-REVIEWCopyright © 1999 by Princeton Review Management, L.L.C. All Rights Reserved.

section 1question 26

Passage FourSynopsis: The author defends the practice of medicine as a profession rather than a trade. The authorattempts to accomplish this by examining what does and does not constitute what a “profession”: first byexamining and dismissing some perspectives and finally by settling on his own concept of the idea (apublic pronouncement of a commitment to a way of life that serves a higher good).

Question 26: Describe the author’s attitude towards professionals. The answer will be generally positiveand should reflect some of what the author claims are the key attributes of a professional.

(A) The evidence in the passage supports the view that the author is eager that physicians be accordedthe respect that their profession has always commanded, and it could be argued that the author isfighting the idea that physicians be viewed from a new perspective (that new perspective being thatmedicine is a trade rather than a profession).

(B) No mention is made of professionals who have become demoralized by public opinion.(C) The author never expresses any surprise over these events.(D) Whether most professionals have come to be considered technicians is not discussed in the passage.

The only group of professionals that the author discusses are physicians.(E) This is the credited response. The author sees this confessed commitment as a necessary part of the

definition of a professional.

rt LSATPrepTest 1

www.review.com 800-2-REVIEWwww.review.com 800-2-REVIEWCopyright © 1999 by Princeton Review Management, L.L.C. All Rights Reserved.

section 1question 27

Passage FourSynopsis: The author defends the practice of medicine as a profession rather than a trade. The authorattempts to accomplish this by examining what does and does not constitute what a “profession”: first byexamining and dismissing some perspectives and finally by settling on his own concept of the idea (apublic pronouncement of a commitment to a way of life that serves a higher good).

Question 27: The logical beginning of the paragraph that follows the passage would be closely related tothe ideas discussed in the final paragraph of the passage. The final paragraph of the passage settles onthe idea that a professional makes an articulated and public commitment to a way of life that pursues ahigher good. The following paragraph will either expand upon that theme or introduce a counter consider-ation.

(A) This has no relation to the preceding paragraph and consequently would be a bad answer choice.(B) This is the credited response. It is directly related to the themes in the preceding paragraph.(C) This has no relation to the preceding paragraph and consequently would be a bad answer choice.(D) The passage is concerned with defining “profession” in general in an effort to defend the idea that

medical practitioners are professionals. No efforts are made to define the characteristics of the medicalprofession specifically.

(E) The meaning of illness and its potential threat? Sorry.

rt LSATPrepTest 1

www.review.com 800-2-REVIEWwww.review.com 800-2-REVIEWCopyright © 1999 by Princeton Review Management, L.L.C. All Rights Reserved.

section 1question 28

Passage FourSynopsis: The author defends the practice of medicine as a profession rather than a trade. The authorattempts to accomplish this by examining what does and does not constitute what a “profession”: first byexamining and dismissing some perspectives and finally by settling on his own concept of the idea (apublic pronouncement of a commitment to a way of life that serves a higher good).

Question 28: What is the author trying to accomplish between lines 18-42? That’s the part of the passagethat discusses and then dismisses different interpretations in what it means to be a “professional”. This isused to set up the author’s definition of what is a “professional”. This is best captured in answer choice(C).

rt LSATPrepTest 1

www.review.com 800-2-REVIEWwww.review.com 800-2-REVIEWCopyright © 1999 by Princeton Review Management, L.L.C. All Rights Reserved.

section 2question 1

Game OneSix people sitting in six chairs. This should be one of the more do-able games in the section. Start off bydrawing a basic diagram and listing your game pieces:

KLMNOP2

1 3

6 45

Now all that’s left to do is to figure out where the six representatives go. Your next step should be tosymbolize your available clues and draw deductions.

Clue #1: PN or NP Clue #3: KM MK

Clue #2: LM ML or LN NL (or both) Clue #4: OP or PO OM and MO

Deduction (from Clue #4): OM or MO OP and PO

Question 1: You are looking for an arrangement that violates none of the clues. To do this type of ques-tion, take one clue at a time and eliminate answer choices that violate that clue. When you get down to oneremaining choice, that’s the one to pick.

Clue #1: Eliminate choice (E).Clue #2: Eliminate choice (A).Clue #3: Eliminate choice (D).Clue #4: Eliminate choice (C).The only remaining choice is (B).

rt LSATPrepTest 1

www.review.com 800-2-REVIEWwww.review.com 800-2-REVIEWCopyright © 1999 by Princeton Review Management, L.L.C. All Rights Reserved.

section 2question 2

Game OneSix people sitting in six chairs. This should be one of the more do-able games in the section. Start off bydrawing a basic diagram and listing your game pieces:

KLMNOP2

1 3

6 45

Now all that’s left to do is to figure out where the six representatives go. Your next step should be tosymbolize your available clues and draw deductions.

Clue #1: PN or NP Clue #3: KM MK

Clue #2: LM ML or LN NL (or both) Clue #4: OP or PO OM and MO

Deduction (from Clue #4): OM or MO OP and PO

Question 2: If L sits next to P, who else must sit next to each other? If L sits on one side of P, then clue #1should be sufficient to let you know that N sits on the other side of P (LPN). Further, since L sits next toeither M or N (clue #2), M must be sitting on the other side of L (MLPN). Finally, O and K must sit next toeach other since they’re the only two left and it’s a circular table (OKMLPN or KOMLPN). The creditedresponse is therefore (A).

rt LSATPrepTest 1

www.review.com 800-2-REVIEWwww.review.com 800-2-REVIEWCopyright © 1999 by Princeton Review Management, L.L.C. All Rights Reserved.

section 2question 3

Game OneSix people sitting in six chairs. This should be one of the more do-able games in the section. Start off bydrawing a basic diagram and listing your game pieces:

KLMNOP2

1 3

6 45

Now all that’s left to do is to figure out where the six representatives go. Your next step should be tosymbolize your available clues and draw deductions.

Clue #1: PN or NP Clue #3: KM MK

Clue #2: LM ML or LN NL (or both) Clue #4: OP or PO OM and MO

Deduction (from Clue #4): OM or MO OP and PO

Question 3: If K sits between L and P, M sits between . . . You have LKP, and clue #1 tells you that P andN must be next to each other (LKPN). Further, clue #2 tells you that L must sit next to either M or N, and Nhas already been placed. Therefore, now you have MLKPN. Since O is the only one left, and since it’s acircular table, go ahead and place O on either end of the line (OMLKPN). It should now be pretty clear thatM sits between L and O, and the credited response is therefore (B).

rt LSATPrepTest 1

www.review.com 800-2-REVIEWwww.review.com 800-2-REVIEWCopyright © 1999 by Princeton Review Management, L.L.C. All Rights Reserved.

section 2question 4

Game OneSix people sitting in six chairs. This should be one of the more do-able games in the section. Start off bydrawing a basic diagram and listing your game pieces:

KLMNOP2

1 3

6 45

Now all that’s left to do is to figure out where the six representatives go. Your next step should be tosymbolize your available clues and draw deductions.

Clue #1: PN or NP Clue #3: KM MK

Clue #2: LM ML or LN NL (or both) Clue #4: OP or PO OM and MO

Deduction (from Clue #4): OM or MO OP and PO

Question 4: If N sits next to M, K can sit between . . . Start by referring to clue #1, which gives you PNM.Clue #2 gets you to PNML. The two representatives left are O and K, and there are two possible arrange-ments: PNMLOK or PNMLKO. So K sits either between O and P or between O and L. The answer istherefore (E).

rt LSATPrepTest 1

www.review.com 800-2-REVIEWwww.review.com 800-2-REVIEWCopyright © 1999 by Princeton Review Management, L.L.C. All Rights Reserved.

section 2question 5

Game OneSix people sitting in six chairs. This should be one of the more do-able games in the section. Start off bydrawing a basic diagram and listing your game pieces:

KLMNOP2

1 3

6 45

Now all that’s left to do is to figure out where the six representatives go. Your next step should be tosymbolize your available clues and draw deductions.

Clue #1: PN or NP Clue #3: KM MK

Clue #2: LM ML or LN NL (or both) Clue #4: OP or PO OM and MO

Deduction (from Clue #4): OM or MO OP and PO

Question 5: You can use earlier work to help solve this problem. From question 2, L sits next to M and L isalso next to P. Eliminate (A) and (B) since they don’t include P. From question 3, L sits next to M and L isalso next to K. Eliminate (C) because it doesn’t include K. Next, notice how the remaining two choices areonly different in that (E) has L sitting next to N and (D) does not (K, O, and P must all work because theyare listed in all remaining answer choices). So what you need to do now is try to come up with an arrange-ment that has L sitting between N and M (you are trying to prove that (E) can work; if you can’t make itwork then you’ll pick (D)). PNLMOK is an arrangement that satisfies all the conditions of the game, so itcan be true that N sits next to L when L is also next to M. The credited response is therefore (E).

rt LSATPrepTest 1

www.review.com 800-2-REVIEWwww.review.com 800-2-REVIEWCopyright © 1999 by Princeton Review Management, L.L.C. All Rights Reserved.

section 2question 6

Game OneSix people sitting in six chairs. This should be one of the more do-able games in the section. Start off bydrawing a basic diagram and listing your game pieces:

KLMNOP2

1 3

6 45

Now all that’s left to do is to figure out where the six representatives go. Your next step should be tosymbolize your available clues and draw deductions.

Clue #1: PN or NP Clue #3: KM MK

Clue #2: LM ML or LN NL (or both) Clue #4: OP or PO OM and MO

Deduction (from Clue #4): OM or MO OP and PO

Question 6: This is another question that you can use earlier work to help solve. From the example in theproblem above (PNLMOK), you can demonstrate that (A), (B), (D), and (E) can be true. Since you arelooking for something that must be false, (C) must be the credited response.

rt LSATPrepTest 1

www.review.com 800-2-REVIEWwww.review.com 800-2-REVIEWCopyright © 1999 by Princeton Review Management, L.L.C. All Rights Reserved.

section 2question 7

Game OneSix people sitting in six chairs. This should be one of the more do-able games in the section. Start off bydrawing a basic diagram and listing your game pieces:

KLMNOP2

1 3

6 45

Now all that’s left to do is to figure out where the six representatives go. Your next step should be tosymbolize your available clues and draw deductions.

Clue #1: PN or NP Clue #3: KM MK

Clue #2: LM ML or LN NL (or both) Clue #4: OP or PO OM and MO

Deduction (from Clue #4): OM or MO OP and PO

Question 7: Again, start by using previous work to eliminate answer choices. Question #1 allows you toeliminate choice (A), question #2 allows you to eliminate choice (D), and question #6 allows you toeliminate choice (C). The answer must therefore be either (B) or (E). By trying one of these you can get theanswer (either it can work and you pick the other one or it can’t work and then you’ve found the answer –remember, you’re looking for something that CANNOT be true. By trying (B), you’ll start with NLKO (K isnext to O and L is between K and N). Next, clue #1 allows you to place P next to N (PNLKO). Finally, sinceM is the only one left you can tack it on to one end or the other (remember, it’s a circular table so it doesn’tmatter which end). What you have now is MPNLKO. This arrangement satisfies all conditions so you’llwant to pick (E). To confirm, choice (E) starts out with KOLN. Clue #1 gets you to KOLNP, and then it’sjust an issue of tacking M on one end or the other (again, it’s a circular table so it doesn’t really matterwhich end). So you’re left with MKOLNP. This violates clue #3, and that’s why (E) is the credited re-sponse.

rt LSATPrepTest 1

www.review.com 800-2-REVIEWwww.review.com 800-2-REVIEWCopyright © 1999 by Princeton Review Management, L.L.C. All Rights Reserved.

section 2question 8

Game TwoStart off by drawing a basic diagram. Keep your printers and computers on separate lines.

1 2 3 4 COMPUTER 1988 1988/1989 1987/1988 1987/1988 PRINTER 1988/1989 1988/1989 1988 1987/1988

Clue #1: The computer in each room is either the same age or older than the printer. Mark this clue so youdon’t forget to use it.Clue #2: Mark this clue into your diagram (it’s been done above already).Clue #3: Mark this clue into your diagram (it’s been done above already).Clue #4: Mark this clue into your diagram (it’s been done above already).Clue #5: Mark this information into your diagram (it’s been done above already).

Deductions: Start by narrowing possibilities by applying clue #1. That work is reflected above.

Question 8: If the computer in office 3 was purchased in 1987 (deduction), what else could be true? Fromthis information, you can also deduce that both the computer and the printer in office 4 were purchased in1987. There is plenty of stuff going on in offices 1 and 2 that isn’t set, and that’s probably where theanswer to this question will lie.

(A) This can never be true.(B) This is the credited response.(C) For this question, you can determine that the computer in office 4 was purchased in 1987.(D) For this question, you can determine that the printer in office 4 was purchased in 1987.(E) For this question, you can determine that the printer in office 4 was purchased in 1987.

rt LSATPrepTest 1

www.review.com 800-2-REVIEWwww.review.com 800-2-REVIEWCopyright © 1999 by Princeton Review Management, L.L.C. All Rights Reserved.

section 2question 9

Game TwoStart off by drawing a basic diagram. Keep your printers and computers on separate lines.

1 2 3 4 COMPUTER 1988 1988/1989 1987/1988 1987/1988 PRINTER 1988/1989 1988/1989 1988 1987/1988

Clue #1: The computer in each room is either the same age or older than the printer. Mark this clue so youdon’t forget to use it.Clue #2: Mark this clue into your diagram (it’s been done above already).Clue #3: Mark this clue into your diagram (it’s been done above already).Clue #4: Mark this clue into your diagram (it’s been done above already).Clue #5: Mark this information into your diagram (it’s been done above already).

Deductions: Start by narrowing possibilities by applying clue #1. That work is reflected above.

Question 9: This question can be answered from your original diagram.

(A) No, the printer in office 1 was purchased in either 1988 or 1989.(B) No, the computer in office 2 was purchased in either 1988 or 1989.(C) No, the computer in office 3 was purchased in either 1987 or 1988.(D) This is the credited response. That printer could have been purchased in either 1987 or 1988.(E) No, the printer in office 4 was purchased in either 1987 or 1988.

rt LSATPrepTest 1

www.review.com 800-2-REVIEWwww.review.com 800-2-REVIEWCopyright © 1999 by Princeton Review Management, L.L.C. All Rights Reserved.

section 2question 10

Game TwoStart off by drawing a basic diagram. Keep your printers and computers on separate lines.

1 2 3 4 COMPUTER 1988 1988/1989 1987/1988 1987/1988 PRINTER 1988/1989 1988/1989 1988 1987/1988

Clue #1: The computer in each room is either the same age or older than the printer. Mark this clue so youdon’t forget to use it.Clue #2: Mark this clue into your diagram (it’s been done above already).Clue #3: Mark this clue into your diagram (it’s been done above already).Clue #4: Mark this clue into your diagram (it’s been done above already).Clue #5: Mark this information into your diagram (it’s been done above already).

Deductions: Start by narrowing possibilities by applying clue #1. That work is reflected above.

Question 10: What is the least number of machines that could have been purchased in 1987? Refer to youroriginal diagram and note that you could get away with having no machines purchased in 1987. Thecredited response is therefore (A).

rt LSATPrepTest 1

www.review.com 800-2-REVIEWwww.review.com 800-2-REVIEWCopyright © 1999 by Princeton Review Management, L.L.C. All Rights Reserved.

section 2question 11

Game TwoStart off by drawing a basic diagram. Keep your printers and computers on separate lines.

1 2 3 4 COMPUTER 1988 1988/1989 1987/1988 1987/1988 PRINTER 1988/1989 1988/1989 1988 1987/1988

Clue #1: The computer in each room is either the same age or older than the printer. Mark this clue so youdon’t forget to use it.Clue #2: Mark this clue into your diagram (it’s been done above already).Clue #3: Mark this clue into your diagram (it’s been done above already).Clue #4: Mark this clue into your diagram (it’s been done above already).Clue #5: Mark this information into your diagram (it’s been done above already).

Deductions: Start by narrowing possibilities by applying clue #1. That work is reflected above.

Question 11: This question gives you information that will allow you to make a handful of deductions. Ifthe computer in office 4 was purchased in 1988, clue #1 allows you to deduce that the printer in office 4was also purchased in 1988. If the printer in office 4 was purchased in 1988, clue #3 allows you to deducethat the computer in office 3 was also purchased in 1988. If the computer in office 3 was purchased in 1988,clue #4 allows you to deduce that the computer in office 2 was purchased in 1989. If the computer in office2 was purchased in 1989, clue #2 allows you to deduce that the printer in office 1 was also purchased in1989. Finally, clue #1 allows you to deduce that the printer in office 2 was purchased in 1989. The onlyanswer choice that fits this chain of reasoning is answer choice (B).

rt LSATPrepTest 1

www.review.com 800-2-REVIEWwww.review.com 800-2-REVIEWCopyright © 1999 by Princeton Review Management, L.L.C. All Rights Reserved.

section 2question 12

Game TwoStart off by drawing a basic diagram. Keep your printers and computers on separate lines.

1 2 3 4 COMPUTER 1988 1988/1989 1987/1988 1987/1988 PRINTER 1988/1989 1988/1989 1988 1987/1988

Clue #1: The computer in each room is either the same age or older than the printer. Mark this clue so youdon’t forget to use it.Clue #2: Mark this clue into your diagram (it’s been done above already).Clue #3: Mark this clue into your diagram (it’s been done above already).Clue #4: Mark this clue into your diagram (it’s been done above already).Clue #5: Mark this information into your diagram (it’s been done above already).

Deductions: Start by narrowing possibilities by applying clue #1. That work is reflected above.

Question 12: This question also gives you information that will allow you to make a handful of deductions.You should note, however, that this question is asking for something that could be true. This generallymeans that the answer will involve a part of the diagram where there is still more than one possible solu-tion. If the computer in office 3 was purchased in 1988, then the printer in office 4 was also purchased in1988 (clue #3). Additionally, the computer in office 2 was purchased in 1989 (clue #4) as was the printer inoffice 1 (clue #2). Finally, clue #1 allows you to deduce that the printer in office 2 was purchased in 1989.The only slot where you don’t have a clear idea of an exact year of purchase is the computer in office 4(which could have been purchased in either 1987 or 1988). Again, since this is a could question, youshould expect that the credited response will involve the computer in office 4, and by now answer choice(D) should be looking pretty good.

rt LSATPrepTest 1

www.review.com 800-2-REVIEWwww.review.com 800-2-REVIEWCopyright © 1999 by Princeton Review Management, L.L.C. All Rights Reserved.

section 2question 13

Game TwoStart off by drawing a basic diagram. Keep your printers and computers on separate lines.

1 2 3 4 COMPUTER 1988 1988/1989 1987/1988 1987/1988 PRINTER 1988/1989 1988/1989 1988 1987/1988

Clue #1: The computer in each room is either the same age or older than the printer. Mark this clue so youdon’t forget to use it.Clue #2: Mark this clue into your diagram (it’s been done above already).Clue #3: Mark this clue into your diagram (it’s been done above already).Clue #4: Mark this clue into your diagram (it’s been done above already).Clue #5: Mark this information into your diagram (it’s been done above already).

Deductions: Start by narrowing possibilities by applying clue #1. That work is reflected above.

Question 13: This question begins by changing one of the rules. If the computers in offices 2 and 3 werepurchased in the same year, the first thing to do is check your original diagram to narrow your possibilities.Since the computer in office 2 could have been purchased in either 1988 or 1989, and since the computer inoffice 3 could have been purchased in either 1987 or 1988, the only way that you could have purchasedthem both in the same year would be if they were purchased in 1988. By applying clues #2 and #3, you canalso deduce that the printer in office 1 and the printer in office 4 were also purchased in 1988. By checkingyour diagram you should notice that the only remaining space that could possibly be 1989 would be theprinter in office 2. The credited response is therefore (C).

rt LSATPrepTest 1

www.review.com 800-2-REVIEWwww.review.com 800-2-REVIEWCopyright © 1999 by Princeton Review Management, L.L.C. All Rights Reserved.

section 2question 14

Game ThreeThis game requires you to combine a couple basic set-ups. Although you’ll want to draw a rough grid,you should also notice that the variables (the lawyers) are all compared on a relative scale (who was hiredbefore whom). You will combine these statements to form a “branching” diagram. Lawyers hired early willbe listed on the left and lawyers hired later will be listed to the right. The trick to doing this type of game(which incidentally has not appeared on the LSAT since February of 1994) is to avoid writing a variabledown more than once. Instead, you will start to combine information.

Clue #1: H NClue #2: Skip this one for now since it contains neither H nor N.Clue #3: This is your first opportunity to combine information.

H N GJ

Although you are told that both N and J precede G, you are told nothing about their relative order. Further,you don’t have sufficient information to put H and J in a relative order. Think of the arrows as “bungeecords” which could be stretched out long enough to have J precede H.

Revisit Clue #2: You can now work this information into your diagram.H N GK J

Clue #4: Tack this clue onto your existing diagram.O

H N G IK J M

Clues #5 and #6: Tack these clues onto your existing diagram (see above).

To determine the relative order of hiring, simply follow chains of arrows. If two lawyers aren’t on a similarchain of arrows (M and G for example), that means their relative order cannot be determined (they could behired in either order). If you place this branching diagram above the list of years in which the lawyerscould be hired, it should allow you to have a good idea of where things can and cannot go.

OH N G IK J M

61 62 63 64 65 66 67 68

Next, it’s generally helpful to notice who can be hired first (H or K) and who could be hired last (O, I, or M).

Question 14: Find an answer choice that CANNOT be true. Use your initial diagram for help.

(A) As mentioned above, H could be the first one to join the firm.(B) H could join the firm in 1963 after K (1961) and J (1962).(C) This is the credited response. G joins the firm after at least H, N, K, and J. Therefore, the earliest G

could join the firm would be in 1965.(D) M could join the firm in 1964 after K (1961), J (1962), and H (1963).(E) O could join the firm in 1964 after H (1961), N (1962), and K (1963).

rt LSATPrepTest 1

www.review.com 800-2-REVIEWwww.review.com 800-2-REVIEWCopyright © 1999 by Princeton Review Management, L.L.C. All Rights Reserved.

section 2question 15

Game ThreeThis game requires you to combine a couple basic set-ups. Although you’ll want to draw a rough grid,you should also notice that the variables (the lawyers) are all compared on a relative scale (who was hiredbefore whom). You will combine these statements to form a “branching” diagram. Lawyers hired early willbe listed on the left and lawyers hired later will be listed to the right. The trick to doing this type of game(which incidentally has not appeared on the LSAT since February of 1994) is to avoid writing a variabledown more than once. Instead, you will start to combine information.

Clue #1: H NClue #2: Skip this one for now since it contains neither H nor N.Clue #3: This is your first opportunity to combine information.

H N GJ

Although you are told that both N and J precede G, you are told nothing about their relative order. Further,you don’t have sufficient information to put H and J in a relative order. Think of the arrows as “bungeecords” which could be stretched out long enough to have J precede H.

Revisit Clue #2: You can now work this information into your diagram.H N GK J

Clue #4: Tack this clue onto your existing diagram.O

H N G IK J M

Clues #5 and #6: Tack these clues onto your existing diagram (see above).

To determine the relative order of hiring, simply follow chains of arrows. If two lawyers aren’t on a similarchain of arrows (M and G for example), that means their relative order cannot be determined (they could behired in either order). If you place this branching diagram above the list of years in which the lawyerscould be hired, it should allow you to have a good idea of where things can and cannot go.

OH N G IK J M

61 62 63 64 65 66 67 68

Next, it’s generally helpful to notice who can be hired first (H or K) and who could be hired last (O, I, or M).

Question 15: If J joins the firm in 1962, which answer CANNOT be true? Since you know that K precedesJ, then you can conclude that K joins the firm in 1961. Use your initial diagram for more help.

(A) This certainly could be true. In this example, either H or M joins the firm in 1963.(B) This certainly could be true. In this example, either H or M joins the firm in 1963.(C) If M joins the firm in 1963 (which could happen), then H would have to join the firm in 1964.(D) This certainly could be true. If H joins the firm in 1963 (which could happen), then N could join the

firm in 1964.(E) This is the credited response. The earliest O could join the firm in this question would be in 1965

(after H (1963) and N (1964)).

rt LSATPrepTest 1

www.review.com 800-2-REVIEWwww.review.com 800-2-REVIEWCopyright © 1999 by Princeton Review Management, L.L.C. All Rights Reserved.

section 2question 16

Game ThreeThis game requires you to combine a couple basic set-ups. Although you’ll want to draw a rough grid,you should also notice that the variables (the lawyers) are all compared on a relative scale (who was hiredbefore whom). You will combine these statements to form a “branching” diagram. Lawyers hired early willbe listed on the left and lawyers hired later will be listed to the right. The trick to doing this type of game(which incidentally has not appeared on the LSAT since February of 1994) is to avoid writing a variabledown more than once. Instead, you will start to combine information.

Clue #1: H NClue #2: Skip this one for now since it contains neither H nor N.Clue #3: This is your first opportunity to combine information.

H N GJ

Although you are told that both N and J precede G, you are told nothing about their relative order. Further,you don’t have sufficient information to put H and J in a relative order. Think of the arrows as “bungeecords” which could be stretched out long enough to have J precede H.

Revisit Clue #2: You can now work this information into your diagram.H N GK J

Clue #4: Tack this clue onto your existing diagram.O

H N G IK J M

Clues #5 and #6: Tack these clues onto your existing diagram (see above).

To determine the relative order of hiring, simply follow chains of arrows. If two lawyers aren’t on a similarchain of arrows (M and G for example), that means their relative order cannot be determined (they could behired in either order). If you place this branching diagram above the list of years in which the lawyerscould be hired, it should allow you to have a good idea of where things can and cannot go.

OH N G IK J M

61 62 63 64 65 66 67 68

Next, it’s generally helpful to notice who can be hired first (H or K) and who could be hired last (O, I, or M).

Question 16: What is the latest year in which J could join the firm? J must have joined the firm before G, I,and M (but could have joined after everyone else). Therefore, the latest year that J could join the firmwould be 1965. The credited response is therefore (D).

rt LSATPrepTest 1

www.review.com 800-2-REVIEWwww.review.com 800-2-REVIEWCopyright © 1999 by Princeton Review Management, L.L.C. All Rights Reserved.

section 2question 17

Game ThreeThis game requires you to combine a couple basic set-ups. Although you’ll want to draw a rough grid,you should also notice that the variables (the lawyers) are all compared on a relative scale (who was hiredbefore whom). You will combine these statements to form a “branching” diagram. Lawyers hired early willbe listed on the left and lawyers hired later will be listed to the right. The trick to doing this type of game(which incidentally has not appeared on the LSAT since February of 1994) is to avoid writing a variabledown more than once. Instead, you will start to combine information.

Clue #1: H NClue #2: Skip this one for now since it contains neither H nor N.Clue #3: This is your first opportunity to combine information.

H N GJ

Although you are told that both N and J precede G, you are told nothing about their relative order. Further,you don’t have sufficient information to put H and J in a relative order. Think of the arrows as “bungeecords” which could be stretched out long enough to have J precede H.

Revisit Clue #2: You can now work this information into your diagram.H N GK J

Clue #4: Tack this clue onto your existing diagram.O

H N G IK J M

Clues #5 and #6: Tack these clues onto your existing diagram (see above).

To determine the relative order of hiring, simply follow chains of arrows. If two lawyers aren’t on a similarchain of arrows (M and G for example), that means their relative order cannot be determined (they could behired in either order). If you place this branching diagram above the list of years in which the lawyerscould be hired, it should allow you to have a good idea of where things can and cannot go.

OH N G IK J M

61 62 63 64 65 66 67 68

Next, it’s generally helpful to notice who can be hired first (H or K) and who could be hired last (O, I, or M).

Question 17: If O joins the firm in 1965 and M joins in 1967, how many other partners can be placed forcertain? You know that G and I join the firm after H, N, K, and J. Further, you know for certain that G joinsthe firm before I. Therefore, G joins the firm in 1966 and I joins the firm in 1968. The remainder of thepartners cannot be placed without absolute certainty, because although you know that H precedes N andK precedes J, you cannot determine the relative positions of H and N versus K and J. The creditedresponse is therefore (B). You may be tempted to pick (D) because you can determine the hiring year offour partners, but this selection would be based on misreading the question (which asks how many of theother partners could be definitively placed).

rt LSATPrepTest 1

www.review.com 800-2-REVIEWwww.review.com 800-2-REVIEWCopyright © 1999 by Princeton Review Management, L.L.C. All Rights Reserved.

section 2question 18

Game ThreeThis game requires you to combine a couple basic set-ups. Although you’ll want to draw a rough grid,you should also notice that the variables (the lawyers) are all compared on a relative scale (who was hiredbefore whom). You will combine these statements to form a “branching” diagram. Lawyers hired early willbe listed on the left and lawyers hired later will be listed to the right. The trick to doing this type of game(which incidentally has not appeared on the LSAT since February of 1994) is to avoid writing a variabledown more than once. Instead, you will start to combine information.

Clue #1: H NClue #2: Skip this one for now since it contains neither H nor N.Clue #3: This is your first opportunity to combine information.

H N GJ

Although you are told that both N and J precede G, you are told nothing about their relative order. Further,you don’t have sufficient information to put H and J in a relative order. Think of the arrows as “bungeecords” which could be stretched out long enough to have J precede H.

Revisit Clue #2: You can now work this information into your diagram.H N GK J

Clue #4: Tack this clue onto your existing diagram.O

H N G IK J M

Clues #5 and #6: Tack these clues onto your existing diagram (see above).

To determine the relative order of hiring, simply follow chains of arrows. If two lawyers aren’t on a similarchain of arrows (M and G for example), that means their relative order cannot be determined (they could behired in either order). If you place this branching diagram above the list of years in which the lawyerscould be hired, it should allow you to have a good idea of where things can and cannot go.

OH N G IK J M

61 62 63 64 65 66 67 68

Next, it’s generally helpful to notice who can be hired first (H or K) and who could be hired last (O, I, or M).

Question 18: If O joins before M, what’s the earliest year in which M could have joined. M is nowdefinitely hired after K, J, H, N, and O. This means that the earliest year in which M could have joined thefirm is 1966 (M could still have been hired before G and I). The credited response is therefore (D).

rt LSATPrepTest 1

www.review.com 800-2-REVIEWwww.review.com 800-2-REVIEWCopyright © 1999 by Princeton Review Management, L.L.C. All Rights Reserved.

section 2question 19

Game FourThis game involves assigning colored tickets to six slots (each slot consists of a month and a line number).Your core diagram should look something like this . . .

Line 1 Line 2 Line 3January (r, p, g/y) pFebruary (g, r, y) g in one of these

Clue #2: Circle this clue or make some kind of note next to your diagram to remind you to do this.Clue #3: Don’t use the same color more than once for a given month.Clue #4: Make a note on your diagram (it’s been done already above).Clue #5: Make a note on your diagram (it’s been done already above).Clue #6: This has also been noted in the diagram.Clue #7: If no February tickets are purple, then the February tickets are green, red, and yellow (see Clue

#3). This has also been noted in the diagram above.

Question 19: If the line three tickets for January are red, what can you determine for sure? According toclue #6, if the line three tickets for January are not green then the line three tickets for February must begreen. The credited response is therefore (E).

rt LSATPrepTest 1

www.review.com 800-2-REVIEWwww.review.com 800-2-REVIEWCopyright © 1999 by Princeton Review Management, L.L.C. All Rights Reserved.

section 2question 20

Game FourThis game involves assigning colored tickets to six slots (each slot consists of a month and a line number).Your core diagram should look something like this . . .

Line 1 Line 2 Line 3January (r, p, g/y) pFebruary (g, r, y) g in one of these

Clue #2: Circle this clue or make some kind of note next to your diagram to remind you to do this.Clue #3: Don’t use the same color more than once for a given month.Clue #4: Make a note on your diagram (it’s been done already above).Clue #5: Make a note on your diagram (it’s been done already above).Clue #6: This has also been noted in the diagram.Clue #7: If no February tickets are purple, then the February tickets are green, red, and yellow (see Clue

#3). This has also been noted in the diagram above.

Question 20: If one set of line 2 tickets is green, what else do you know for sure? The line two tickets thatare green must be the February tickets (the January tickets are purple). Next, by combining clues #3 and#6, you can deduce that the line 3 tickets for January are green (no other February tickets besides line 2can be green, and one set of line 3 tickets must be green). Next, you know from clue #4 that one set ofJanuary tickets must be red, and since line 2 is assigned purple tickets and line 3 is assigned green tickets,then the line 1 tickets for January must be red. The credited response is therefore (A).

rt LSATPrepTest 1

www.review.com 800-2-REVIEWwww.review.com 800-2-REVIEWCopyright © 1999 by Princeton Review Management, L.L.C. All Rights Reserved.

section 2question 21

Game FourThis game involves assigning colored tickets to six slots (each slot consists of a month and a line number).Your core diagram should look something like this . . .

Line 1 Line 2 Line 3January (r, p, g/y) pFebruary (g, r, y) g in one of these

Clue #2: Circle this clue or make some kind of note next to your diagram to remind you to do this.Clue #3: Don’t use the same color more than once for a given month.Clue #4: Make a note on your diagram (it’s been done already above).Clue #5: Make a note on your diagram (it’s been done already above).Clue #6: This has also been noted in the diagram.Clue #7: If no February tickets are purple, then the February tickets are green, red, and yellow (see Clue

#3). This has also been noted in the diagram above.

Question 21: You should be able to answer this question from your original setup.

(A) This is the credited response. The January tickets could be red, purple, and yellow.(B) This cannot be true. The February tickets are green, red, and yellow.(C) Since each month must use red tickets, there is no way to restrict their use to only one line (that would

violate clue #2).(D) The January tickets are red, purple, and either green or yellow. This answer choice cannot be true.(E) This cannot be true. See clues #6 and #7 for more information.

rt LSATPrepTest 1

www.review.com 800-2-REVIEWwww.review.com 800-2-REVIEWCopyright © 1999 by Princeton Review Management, L.L.C. All Rights Reserved.

section 2question 22

Game FourThis game involves assigning colored tickets to six slots (each slot consists of a month and a line number).Your core diagram should look something like this . . .

Line 1 Line 2 Line 3January (r, p, g/y) pFebruary (g, r, y) g in one of these

Clue #2: Circle this clue or make some kind of note next to your diagram to remind you to do this.Clue #3: Don’t use the same color more than once for a given month.Clue #4: Make a note on your diagram (it’s been done already above).Clue #5: Make a note on your diagram (it’s been done already above).Clue #6: This has also been noted in the diagram.Clue #7: If no February tickets are purple, then the February tickets are green, red, and yellow (see Clue

#3). This has also been noted in the diagram above.

Question 22: You should be able to answer this question from your original setup.

(A) This would cause you to either violate clue #3 or clue #5.(B) This is the credited response. The January tickets could be yellow, purple, and red for lines 1, 2, and 3

respectively and the February tickets could be red, yellow, and green.(C) If the line 1 tickets for January are yellow, then the line 3 tickets for January would have to be red (clue

#4); however, this would violate clue #5 because the January tickets for line 3 would be red and theline 3 tickets for February would be yellow.

(D) This would cause you to violate either clue #3 (because you would have two sets of green tickets inJanuary) or clue #5 (you would have no green tickets for line 3).

(E) If the line 3 tickets for January were yellow, then the line 1 tickets for January would have to be red(clue #4). This would then violate clue #2 (because the answer choice tells you to make the line 1tickets for February red).

rt LSATPrepTest 1

www.review.com 800-2-REVIEWwww.review.com 800-2-REVIEWCopyright © 1999 by Princeton Review Management, L.L.C. All Rights Reserved.

section 2question 23

Game FourThis game involves assigning colored tickets to six slots (each slot consists of a month and a line number).Your core diagram should look something like this . . .

Line 1 Line 2 Line 3January (r, p, g/y) pFebruary (g, r, y) g in one of these

Clue #2: Circle this clue or make some kind of note next to your diagram to remind you to do this.Clue #3: Don’t use the same color more than once for a given month.Clue #4: Make a note on your diagram (it’s been done already above).Clue #5: Make a note on your diagram (it’s been done already above).Clue #6: This has also been noted in the diagram.Clue #7: If no February tickets are purple, then the February tickets are green, red, and yellow (see Clue

#3). This has also been noted in the diagram above.

Question 23: If the line 3 tickets for February are yellow, clue #5 requires you to make the line 3 tickets forJanuary green. Next, by applying clue #4 you can determine that the line 1 tickets for January are red.Next, apply clue #2 to determine that the line 1 tickets for February are green, and finally since there is onlyone color ticket left you can use for February (don’t forget clue #7), the line 2 tickets for February must bered. Now you have to find the answer choice that either could be false or must be false.

(A) This must be true. The line three tickets for January are green.(B) This must be true. The line one tickets for January are red.(C) This must be true. The line two tickets for February are red.(D) This must be true. See (B) and (C).(E) This is the credited response. There is only one set of yellow tickets (line three tickets for February).

rt LSATPrepTest 1

www.review.com 800-2-REVIEWwww.review.com 800-2-REVIEWCopyright © 1999 by Princeton Review Management, L.L.C. All Rights Reserved.

section 2question 24

Game FourThis game involves assigning colored tickets to six slots (each slot consists of a month and a line number).Your core diagram should look something like this . . .

Line 1 Line 2 Line 3January (r, p, g/y) pFebruary (g, r, y) g in one of these

Clue #2: Circle this clue or make some kind of note next to your diagram to remind you to do this.Clue #3: Don’t use the same color more than once for a given month.Clue #4: Make a note on your diagram (it’s been done already above).Clue #5: Make a note on your diagram (it’s been done already above).Clue #6: This has also been noted in the diagram.Clue #7: If no February tickets are purple, then the February tickets are green, red, and yellow (see Clue

#3). This has also been noted in the diagram above.

Question 24: This question is a little tougher because the test writers have removed a rule. What you areleft with is three colors (red, green, yellow) and you know that you can’t use a color more than once perline (clue #2) or more than once per month per month (clue #3).

(A) The January tickets are red, green, and yellow (in no particular order). In any event, it can never betrue that no January tickets are green.

(B) See (A), but replace the word “January” with “February”.(C) This is the credited response. Your January tickets could be yellow, red, and green for lines 1, 2, and 3

respectively and your February tickets could be green, yellow, and red.(D) This would mean that both sets of line 3 tickets would be yellow, and that’s a violation of clue #2.(E) This would mean that both sets of line 1 tickets would be red, and that’s a violation of clue #2.

rt LSATPrepTest 1

www.review.com 800-2-REVIEWwww.review.com 800-2-REVIEWCopyright © 1999 by Princeton Review Management, L.L.C. All Rights Reserved.

section 3question 1

Question 1: Explain why treating deep wounds with sugar would be an effective treatment for eliminatingbacteria.

(A) This is the credited response. The sugar is an effective use against bacteria because it affects thebacteria’s environment in a way that is detrimental to the bacteria.

(B) Do these treatments include use on deep wounds? Further, this answer does nothing to explain whysugar is effective for the use described in the argument.

(C) We’re looking for a reason that sugar is an effective preventative to the spread of bacteria in a deepwound. This answer makes it even harder to explain that phenomenon.

(D) This has nothing to do with packing a deep wound with sugar.(E) This does nothing to explain how the sugar works towards the end described.

rt LSATPrepTest 1

www.review.com 800-2-REVIEWwww.review.com 800-2-REVIEWCopyright © 1999 by Princeton Review Management, L.L.C. All Rights Reserved.

section 3question 2

Question 2: Parallel the argument. In general terms, the argument proceeds as follows: 1) If A then B. 2)X is a B. 3) Therefore, X is also an A.

This argument is flawed, and although you may be tempted to correct the reasoning, your task is toidentify the argument in the answer choices that is flawed in the same way.

(A) The reasoning in this argument is correct, so you know it’s not the credited response.(B) This is the credited response. Just as there may be other explanations that Gerald cannot distinguish

between green and brown (he may be blind), there may be other reasons that Mary has lost her senseof smell.

(C) The reasoning in this argument is correct, so you know it’s not the credited response.(D) The reasoning in this argument is correct, so you know it’s not the credited response.(E) You can eliminate this answer choice because it brings up a three terms (diabetic, large amounts of

sugar, special diet), while the original argument sticks to two (distinguish between green and brown,red/green color blind).

rt LSATPrepTest 1

www.review.com 800-2-REVIEWwww.review.com 800-2-REVIEWCopyright © 1999 by Princeton Review Management, L.L.C. All Rights Reserved.

section 3question 3

Question 3: Find a general explanation that describes the specific phenomenon in the argument. In thiscase, as postulate is accepted not because scientists understand the underlying forces but because theyhave been able to observe the phenomenon the postulate describes (scientists believe in the idea ofcontinental drift because they have been able to observe it, they still don’t know how it works).

(A) This single example is hardly sufficient to say anything about the aim of science in general.(B) This example doesn’t seem related to a mathematical description of nature.(C) In this example, the improvement in measuring instruments has made a theory easier to work out (it has

been confirmed through direct observation).(D) The concern of science in general cannot be addressed by this single example.(E) This is the credited response. Continental drift was detected due to an improvement in measurement

devices (the events a theory postulates are (were) detected) and the theory (continental drift) wasaccepted even though it is still a mystery as to exactly how continental drift occurs (we have notpinpointed such a force).

rt LSATPrepTest 1

www.review.com 800-2-REVIEWwww.review.com 800-2-REVIEWCopyright © 1999 by Princeton Review Management, L.L.C. All Rights Reserved.

section 3question 4

Question 4: Identify something you can conclusively defend given the statements in the passage.

(A) This statement goes too far. There may be other ways to deter a would-be aggressor nation fromattacking (a substantial cash payment, for example).

(B) According to the argument, a would-be aggressor is deterred based on its assessment of the retalia-tory capabilities of the would-be victim of the aggression. At no time are the retaliatory capabilities oftwo nations compared to each other.

(C) This statement goes too far. There may be other reasons that the nation isn’t attacking (it may just bea peaceful nation).

(D) This is the credited response. Deterrence is based upon other countries’ evaluation of a nation’sretaliatory power. Because of this, it wouldn’t be a good idea to keep one’s retaliatory power a secret.

(E) This statement goes too far. Although a substantial retaliatory threat is necessary, the argumentdoesn’t claim that the threat needs to exceed every other nation’s retaliatory potential. Being able tocompletely destroy an enemy with a retaliatory strike is, according to the author, sufficient. Whetheranother nation has the capability to destroy the planet a hundred times over isn’t relevant.

rt LSATPrepTest 1

www.review.com 800-2-REVIEWwww.review.com 800-2-REVIEWCopyright © 1999 by Princeton Review Management, L.L.C. All Rights Reserved.

section 3question 5

Question 5: Weaken Clay Moltz’s argument. Clay argues that since no planets have been discoveredoutside our solar system, life as we know it is restricted to the planet Earth. The flaw in this argument isthat Clay confuses the idea that we haven’t found something (planets outside the solar system) with thefact that this something doesn’t exist at all.

(A) The argument is restricted to life as we know it. Life other than life as we know it is not relevant to theargument.

(B) This fact would strengthen Clay’s argument.(C) This is the credited response. The fact that we have not found other planets can be explained in a

way other than the fact that other planets do not exist.(D) Clay believes this to be true and feels that one of the argument’s assumptions (other solar systems

contain planets) is an unwarranted assumption.(E) The issue of communication between us and extraterrestrial civilizations is not relevant to the argu-

ment. The argument is only about whether life as we know it exists on other planets.

rt LSATPrepTest 1

www.review.com 800-2-REVIEWwww.review.com 800-2-REVIEWCopyright © 1999 by Princeton Review Management, L.L.C. All Rights Reserved.

section 3question 6

Question 6: Parallel the reasoning. This argument consists of a conditional statement and the properapplication of its contrapositive (if A then B; not B, therefore not A).

(A) This is the credited response. See the above logical structure for more guidance.(B) The structure of this argument is – A then B; B, therefore A. This is a misapplication of the

contrapositive.(C) This is the same structure as answer choice (B).(D) The structure of this argument is – A then B; not A, therefore not B. This is not parallel to the original

argument.(E) The structure of this argument is – A then B; A, therefore B. Although this is a sound argument, it is

not parallel to the original argument.

rt LSATPrepTest 1

www.review.com 800-2-REVIEWwww.review.com 800-2-REVIEWCopyright © 1999 by Princeton Review Management, L.L.C. All Rights Reserved.

section 3question 7

Question 7: Inference. You are looking for something that you know to be true given the information inthe passage. Because this passage only discusses proportions, you don’t have sufficient information tomake any conclusions about raw numbers (one lake could have many more ducks than the other lakedoes). This reasoning is enough to enable you to eliminate choices (C), (D), and (E). Next, the argumenttells you that by examining gender rations at each lake you can deduce that the eastern lake contains moreadult male ducks than the western lake does (and that the western lake contains more non-adult male ducksthan the eastern lake does). This should be sufficient to enable you to select answer choice (A).

rt LSATPrepTest 1

www.review.com 800-2-REVIEWwww.review.com 800-2-REVIEWCopyright © 1999 by Princeton Review Management, L.L.C. All Rights Reserved.

section 3question 8

Question 8: Identify the flaw in the argument. Food additives are evaluated by comparing health-relatedbenefits with potential risks. Yellow Dye No. 5 has a potential risk (allergic reactions). The author of theargument counters this claim with a statement that the use of Yellow Dye No. 5 enhances consumers’enjoyment of lemon soda and concludes that Yellow Dye No. 5 should not be banned. Is the enhancedenjoyment of lemon soda a health-related benefit? The author makes this assumption and this is a goodplace to start when looking for a flaw in the argument.

(A) The author claims that Yellow Dye No. 5 might cause allergic reactions in a few consumers. Thisdoesn’t sound much like an implication that the dye entails no health-related risks.

(B) This is the credited response. See the above discussion for more detail.(C) This argument is only about one particular food additive (Yellow Dye No. 5). Other food additives are

not relevant to the argument.(D) The argument (Yellow Dye No. 5 should not be banned) is based on a flimsy assumption that en-

hanced enjoyment of a beverage entails a health-related benefit.(E) Warning labels are not relevant to this argument.

rt LSATPrepTest 1

www.review.com 800-2-REVIEWwww.review.com 800-2-REVIEWCopyright © 1999 by Princeton Review Management, L.L.C. All Rights Reserved.

section 3question 9

Question 9: Weaken the argument. The argument states that businesses will install environmentalsafeguards because the fines levied against businesses responsible for environmental accidents exceedthe costs of adopting appropriate safety measures. The author assumes that businesses are able toaccurately evaluate the potential monetary risk of an environmental accident versus the costs involved inavoiding those accidents. If this assumption is not true, businesses may choose to not adopt safetymeasures because they are mistaken in their evaluation of potential risks.

(A) This is the credited response. If businesses greatly underestimate the risk of future accidents, theymay choose to not install adequate environmental safeguards because it seems to be unreasonablyexpensive when compared to the potential risk of paying a fine.

(B) There is no discussion of short-term versus long-term profit in the argument.(C) If anything, this statement would strengthen the argument.(D) How businesses see fines levied against them is not relevant to the argument.(E) How businesses promote themselves is not relevant to the argument.

rt LSATPrepTest 1

www.review.com 800-2-REVIEWwww.review.com 800-2-REVIEWCopyright © 1999 by Princeton Review Management, L.L.C. All Rights Reserved.

section 3question 10

Question 10: Find the statement that is not necessary for the proposed course of action in the argument tobe implemented. The argument claims that even in a democracy the dissemination of certain advancedtechnological knowledge should be controlled. The purpose of controlling this information is to keep itfrom falling into the hands of other countries that may become competitors or enemies. However, theauthor also states that scientific information should be freely exchanged.

(A) This is necessary. If a nation can’t distinguish one group from the other it would be impossible toeffectively restrict information from falling into the hands of countries that may become competitors orenemies.

(B) This is necessary. If the course of action proposed would render the democratic process ineffective, itshould not be implemented.

(C) This is the credited response. The argument is not concerned with the relative levels of scientificresearch from country to country. The argument is instead concerned with restricting the dissemina-tion of information in certain cases.

(D) This is necessary. The argument states that there should be a free exchange of scientific informationwhile also stating that certain types of technological knowledge ought to be controlled. Without theability to distinguish between the two, information that should be disseminated might not be whileinformation that should be controlled might be disseminated.

(E) This is necessary. In a case where it is necessary to restrict the dissemination of advanced technicalknowledge, a prohibition on disseminating certain information relies on the enforceability of thatprohibition.

rt LSATPrepTest 1

www.review.com 800-2-REVIEWwww.review.com 800-2-REVIEWCopyright © 1999 by Princeton Review Management, L.L.C. All Rights Reserved.

section 3question 11

Question 11: Inference. Find something you know to be true given the information in the argument.

(A) This contradicts information given in the first sentence of the argument.(B) This is the credited response. In interglacial periods, the oxygen-16 removed from the seawater is

returned through precipitation. During an ice age, however, the oxygen-16 removed from the seawateris trapped in ice caps. This leaves a greater concentration of oxygen-18 in the seawater than existsduring interglacial periods.

(C) There is nothing in the argument to suggest that the proportion of oxygen-16 in precipitation is everchanged. The difference is that during an ice age the precipitation is trapped in ice caps.

(D) At no point is the proportion of precipitation falling on land versus sea ever discussed.(E) The argument suggests that during interglacial periods that the composition of seawater doesn’t

change over the long term (the oxygen-16 that is removed from seawater through water vapor isreturned to seawater in the form of precipitation). You can well imagine that it’s hard to change moreslowly than something that doesn’t really change at all.

rt LSATPrepTest 1

www.review.com 800-2-REVIEWwww.review.com 800-2-REVIEWCopyright © 1999 by Princeton Review Management, L.L.C. All Rights Reserved.

section 3question 12

Question 12: Identify a flaw in the argument. The argument states that as the average age of a populationincreases, national savings rates will decline. The author reasons that this is true because older peoplehave fewer reasons to save than do younger people. The author assumes that each reason to save moneywill result in a more or less equal amount of money saved, or the number of reasons someone would haveto save money would not be relevant to the argument. An older person might be saving for retirement (onereason) while a younger person might be saving to buy a new CD and to take a date to dinner (tworeasons). Obviously, the former person would have to save much more money than does the latter person.

(A) It’s not the specific reasons that are important, and it certainly doesn’t matter which reason is thestrongest, it’s the amount of money that will be saved that’s important.

(B) A negative savings rate is not relevant to the argument. What is relevant is the relationship theauthor draws between reasons for saving money and the amount of money saved.

(C) Since this argument is framed as a hypothetical case (if the average age of these nations’ populationscontinues to rise . . .), whether the average age of the population is actually rising isn’t relevant.

(D) This is the credited response. See the above analysis for more detail.(E) Any answer choice that would require this kind of specific knowledge will never be right on the LSAT.

The more apparent flaw is that the author equates reasons for savings with an amount of money thatwill be saved.

rt LSATPrepTest 1

www.review.com 800-2-REVIEWwww.review.com 800-2-REVIEWCopyright © 1999 by Princeton Review Management, L.L.C. All Rights Reserved.

section 3question 13

Question 13: Identify the main point. The author uses the example of German Shepherds that are used aspolice dogs or seeing-eye dogs to illustrate the fact that a “pit bull” is not a breed of dog. The designation“pit bull” instead refers to what the dog does.

(A) Although this statement can be true given the argument, it is not the main point of the argument.(B) The author would likely disagree with this statement. A “pit bull” is what it is because of what it does,

and from the argument it is conceivable that different breeds could be “pit bulls”.(C) This is the credited response. See the discussion above.(D) This is used to illustrate the difference between identifying a dog by its breed versus identifying a dog

by its function. It contributes to the support of the main point.(E) This idea is never mentioned or supported in the argument.

rt LSATPrepTest 1

www.review.com 800-2-REVIEWwww.review.com 800-2-REVIEWCopyright © 1999 by Princeton Review Management, L.L.C. All Rights Reserved.

section 3question 14

Question 14: Inference. Identify an answer choice that must be true given the information in the argument.Note that in this argument that there is a sentence that can be expressed as a conditional statement (ifmonetary system, then marketplace). Next, the contrapositive of that conditional statement should also benoted (if no marketplace, then no monetary system). This should be sufficient information to support thecredited response.

(A) The discussion of Mesopotamian and Greek cities should not be considered an exhaustive discussionof the cultures that existed in the fourth century B.C. There could be other societies with monetarysystems that just aren’t discussed here.

(B) This is backwards. Marketplaces precede monetary systems.(C) Although the argument does state that Mesopotamian cities engaged in trade, it does not say with

whom they were trading.(D) The argument doesn’t address what might have happened after the fourth century B.C.(E) This is the credited response. The argument tells you that Mesopotamian cities never had market-

places. This statement combined with the contrapositive mentioned above would allow you to drawthis conclusion.

rt LSATPrepTest 1

www.review.com 800-2-REVIEWwww.review.com 800-2-REVIEWCopyright © 1999 by Princeton Review Management, L.L.C. All Rights Reserved.

section 3question 15

Questions 15 and 16: Strengthen (15) and Inference (16). To do question 15, you’ll want to start with agood paraphrase of the argument (main point, reasons, assumptions). The point of the argument is thatbusinesses should implement variations in their computer operating system software. This should be donebecause it will inhibit the spread of computer viruses while not creating any loss of computer compatibilityto the business. The author assumes that the implementation of this solution will not cause more drasticproblems for the company as a whole or that failing to implement the solution will cause more drasticproblems for the company as a whole. To strengthen this argument, support one of these assumptions.

Question 15: Strengthen the argument.

(A) The argument only addresses computer compatibility within a business.(B) This is the credited response. The proposed solution is preferable to not having a solution at all.(C) Incompatible operating systems are not relevant to the argument.(D) Other kinds of destructive computer programs are not relevant to the argument.(E) Whether businesses need to share data is unimportant. The argument deals with computers that are

networked (necessarily so or not).

rt LSATPrepTest 1

www.review.com 800-2-REVIEWwww.review.com 800-2-REVIEWCopyright © 1999 by Princeton Review Management, L.L.C. All Rights Reserved.

section 3question 16

Questions 15 and 16: Strengthen (15) and Inference (16). To do question 15, you’ll want to start with agood paraphrase of the argument (main point, reasons, assumptions). The point of the argument is thatbusinesses should implement variations in their computer operating system software. This should be donebecause it will inhibit the spread of computer viruses while not creating any loss of computer compatibilityto the business. The author assumes that the implementation of this solution will not cause more drasticproblems for the company as a whole or that failing to implement the solution will cause more drasticproblems for the company as a whole. To strengthen this argument, support one of these assumptions.

Question 16: Inference. Find something you know to be true given the text of the argument.

(A) Not necessarily. A virus would have to be introduced to the system for this to happen.(B) Not quite. The argument says most of the data will be destroyed.(C) This is the credited response. The computer that the virus is initially introduced to will be damaged

regardless of how the network is configured.(D) Because other potential methods for protecting computers from viruses have not been discussed, we

can’t say for sure whether or not they exist. Consequently, it is impossible to say whether thisstatement is true.

(E) Ease of access to data is never discussed.

rt LSATPrepTest 1

www.review.com 800-2-REVIEWwww.review.com 800-2-REVIEWCopyright © 1999 by Princeton Review Management, L.L.C. All Rights Reserved.

section 3question 17

Question 17: Identify the assumption. The argument makes two central claims – first, a superior conductorhas the authority to insist that rehearsal work must be intensified; second, this authority is earned by theconductor winning the orchestra’s respect for the artistic interpretations the conductor is pursuing. Theauthor is making two main assumptions – first, the conductor cannot win the orchestra’s respect by someother means; second, the orchestra is able to appreciate the conductor’s vision before it has been imple-mented.

(A) This is not important to the argument. Even if the conductor uses the same interpretation with eachorchestra, the conductor still needs to win the respect of at least one orchestra (the first one to playthat particular interpretation) in order to be able to demand intensified rehearsals.

(B) The argument is about the relationship between a superior conductor and an orchestra, not thequalities of superior conductors in general.

(C) The argument states that only a certain subset of conductors can command this kind of respect. Thisassumption is too broad given the argument.

(D) This is the credited response. If this were not true, the second half of the argument wouldn’t makeany sense.

(E) Whether this statement is or isn’t true has no bearing on the argument.

rt LSATPrepTest 1

www.review.com 800-2-REVIEWwww.review.com 800-2-REVIEWCopyright © 1999 by Princeton Review Management, L.L.C. All Rights Reserved.

section 3question 18

Question 18: Resolve the paradox. The paradox is that the amount of oil considered extractable hasremained constant although no new oil fields have been discovered and consumption of domesticallyproduced oil has increased. So why hasn’t the amount of oil considered extractable decreased?

(A) Imported oil is not relevant to the argument unless it’s being put into the ground in the United States.(B) But the consumption has increased while no new oil fields have been discovered. So why hasn’t the

amount of oil considered extractable decreased?(C) This adds nothing to the argument. You have already been told that no new oil fields have been

discovered.(D) The price of oil has no bearing on the paradox.(E) This is the credited response. The oil that has been extracted has been “replaced” by our enhanced

ability to extract oil reserves that we already knew about.

rt LSATPrepTest 1

www.review.com 800-2-REVIEWwww.review.com 800-2-REVIEWCopyright © 1999 by Princeton Review Management, L.L.C. All Rights Reserved.

section 3question 19

Question 19: Identify an assumption. The main point of this argument is that for a railroad to be a suc-cessful business it must focus exclusively on either freight or commuter service. The reason the authorgives is that when a railroad divides its attention between these two types of service, the quality of servicesuffers. The author assumes (note the shift in language from the reason to the point) that “quality ofservice” and “successful business” are necessarily related.

(A) Whether the two types of service have “little” in common is not a necessary assumption. They are ofcourse different enough that service suffers if a railroad tries to focus on both, but that’s not enoughto establish (or require) that the two have “little” in common.

(B) The author never discusses the relative priorities of a railroad, so this is not going to be necessary tothe argument.

(C) This is the credited response. Note how it links the two major terms from the argument.(D) Not necessarily. Although the author claims that if a railroad divides attention between two types of

service it will not be a successful business, the author does not assume that all it takes to be asuccessful business is to focus on one service or another. In more formal terms, “if A, then B” is notequivalent to “if not A, then not B”.

(E) The argument is not related to customer wants, it’s instead based on quality of service. Althoughthey might seem to be the same thing, the two ideas aren’t explicitly the same.

rt LSATPrepTest 1

www.review.com 800-2-REVIEWwww.review.com 800-2-REVIEWCopyright © 1999 by Princeton Review Management, L.L.C. All Rights Reserved.

section 3question 20

Question 20: Inference. Find something you can most strongly support given the information in theargument.

(A) The idea of “the value of business to society” is not addressed in the argument.(B) The argument draws no connection between a business’s efficiency and how the public may perceive

it.(C) The argument draws no connection between how dynamic a business is and how the public may

perceive it.(D) There is nothing in the argument that indicates whether or not these two ideas (providing consumers

with value, being regarded as socially responsible) are related. It’s possible that this statement is true,but it’s not supported by the argument.

(E) This is the credited response. It says in the argument that consumers feel that both large and smallbusinesses provide customers with fairly priced goods and services, yet at the same time big businessis perceived as socially responsible only in times of prosperity. This means that there are times whenbig business is thought to provide customers with fairly priced goods and services yet big business isnot perceived as socially responsible (times that are not prosperous). One can reasonably concludethen that people regard the social responsibility of a big business as something beyond providingfairly priced goods and services.

rt LSATPrepTest 1

www.review.com 800-2-REVIEWwww.review.com 800-2-REVIEWCopyright © 1999 by Princeton Review Management, L.L.C. All Rights Reserved.

section 3question 21

Question 21: Identify an assumption. Some questions on the LSAT are really just not very fun to do. Thisis one of them. There are two major principles described: heavier animals need more energy to moveuphill, animals with larger surface areas have more energy available. So if you’re an animal that wants tomove uphill, your best bet is to be light and have a lot of surface area. The next part of the argument claimsthat squirrels have an easier time moving uphill (or up a tree trunk) than do heavier animals. The fact thatthe author hasn’t discussed surface area at this point means that the author is assuming that ratios ofsurface area to weight are at least comparable for large and small animals (or small animals have a greatersurface area to weight ratio than large animals have).

(A) This contradicts the first sentence in the argument.(B) At no time is relative speed of large versus small animals discussed.(C) This is the credited response. If this weren’t true, large animals would have more energy available to

perform the task of running uphill (or up a tree trunk).(D) Careful. The argument compares body weight to energy output available. This answer choice

compares body weight to actual energy output.(E) Although the author mentions that squirrels can move up trees about as quickly as they move on level

ground, speed is not a factor in this argument.

rt LSATPrepTest 1

www.review.com 800-2-REVIEWwww.review.com 800-2-REVIEWCopyright © 1999 by Princeton Review Management, L.L.C. All Rights Reserved.

section 3question 22

Question 22: Identify the flaw. The author attempts to defend the characterization of the 1980s as beingcharacterized by selfishness by citing human history and the “selfish” concern for the survival of thespecies. The problem with the argument is that the selfishness attributed to the 1980s is behavior thatgoes beyond the pale of the basic survival instincts that the author cites. It is, in effect, two differentsenses of the word “selfish”.

(A) The claim that selfishness has been present throughout history is the argument.(B) The ratio of selfish to unselfish behavior is absolutely irrelevant to the argument.(C) The argument does the opposite of this.(D) Other species are not relevant to the argument.(E) This is the credited response. See the above analysis for more detail.

rt LSATPrepTest 1

www.review.com 800-2-REVIEWwww.review.com 800-2-REVIEWCopyright © 1999 by Princeton Review Management, L.L.C. All Rights Reserved.

section 3question 23

Question 23: Strengthen the argument. A medical journal is making a decision to introduce a new format.They are basing their decision on the results of a survey (62% of the respondents favored the new format).It would help support the argument tremendously if you could demonstrate that the survey respondentsare representative of prospective readership.

(A) This certainly helps to determine that the 62% figure cited is an accurate one, but it fails to establishthat the readers are representative of potential new readers.

(B) What other journals have done is not relevant to the argument.(C) This is the credited response. See the above discussion for more detail.(D) Cost is not relevant to the argument.(E) This would tend to call the survey into question since it is not based on a random sampling of a

population.

rt LSATPrepTest 1

www.review.com 800-2-REVIEWwww.review.com 800-2-REVIEWCopyright © 1999 by Princeton Review Management, L.L.C. All Rights Reserved.

section 3question 24

Questions 24-25: Shanna claims that ownership of a work of art bestows upon the owner the right todestroy that work of art if the owner finds the work of art morally or aesthetically distasteful or if the ownerfeels that it is inconvenient to care for the piece. Jorge counters this claim by asserting a moral argumentthat claims that a unique work of art with aesthetic or historic value belongs to posterity and must bepreserved regardless of the owner’s wishes.

Question 24: Identify an answer choice that would allow Shanna to prevail in her argument against Jorge.You are looking for something that asserts rights of ownership.

(A) This would support Jorge’s argument, if anything.(B) This clearly supports Jorge’s argument.(C) This would support Jorge’s argument, if anything.(D) This clearly supports Jorge’s argument.(E) This is the credited response. Shanna would then only have to make sure that in destroying a work of

art that she isn’t endangering anyone’s health or safety (don’t dynamite a statue in the middle of acrowded schoolyard).

rt LSATPrepTest 1

www.review.com 800-2-REVIEWwww.review.com 800-2-REVIEWCopyright © 1999 by Princeton Review Management, L.L.C. All Rights Reserved.

section 3question 25

Questions 24-25: Shanna claims that ownership of a work of art bestows upon the owner the right todestroy that work of art if the owner finds the work of art morally or aesthetically distasteful or if the ownerfeels that it is inconvenient to care for the piece. Jorge counters this claim by asserting a moral argumentthat claims that a unique work of art with aesthetic or historic value belongs to posterity and must bepreserved regardless of the owner’s wishes.

Question 25: Find a statement where each party has a definite opinion that is in conflict with the otherparty’s opinion.

(A) This is the credited response. Shanna would agree with this statement, but Jorge wouldn’t. To agreethat the owner of the work would be ethically justified in destroying it, Jorge would also have tobelieve that the work was not unique and that the work lacked historic or aesthetic value.

(B) Public viewing is a topic that neither Shanna nor Jorge address.(C) What actually happens with works of art is not addressed by either Shanna or Jorge.(D) Shanna and Jorge would agree on this point because the work is not unique.(E) Shanna and Jorge’s argument is a moral argument. It is not a legal argument.

rt LSATPrepTest 1

www.review.com 800-2-REVIEWwww.review.com 800-2-REVIEWCopyright © 1999 by Princeton Review Management, L.L.C. All Rights Reserved.

section 4question 1

Question 1: Identify a weakness in the argument. The argument states that because we have no effectiveplans for dealing with nuclear waste (including waste that has already been produced), we should shutdown all present nuclear plants and build no new nuclear plants. While this solution would address theissue of future nuclear waste (no more would be produced), it does nothing to address the problem ofexisting nuclear waste.

(A) There is nothing in the argument that would indicate that other means of generating electric powercouldn’t be developed.

(B) Nuclear waste is not strictly linked to plants that operate unsafely, the problem here is nuclear wasteproduced from the normal operation of a nuclear plant.

(C) This is the credited response. See the above analysis.(D) Power generated from fossil fuels is not relevant to the argument.(E) No matter when the problem of dealing with nuclear waste will arrive, it is a problem that must be

addressed completely.

rt LSATPrepTest 1

www.review.com 800-2-REVIEWwww.review.com 800-2-REVIEWCopyright © 1999 by Princeton Review Management, L.L.C. All Rights Reserved.

section 4question 2

Question 2: Inference. Find a statement that must be true given the information in the argument.

(A) This is the credited response. All the fruit fly species now present in Hawaii are thought to bedescendants of the same one or two ancestral females. Since the picture-winged drosophilids are atype of fruit fly, they must also be thought to be descendants of those ancestral females.

(B) The fact that picture-winged drosopholids can be found in Hawaii is not sufficient to prove that theydon’t exist in other places as well.

(C) The fact that the fruit fly species in the Hawaiian archipelago are thought to be descendants of thesame one or two female fruit flies is not enough to prove that all fruit flies worldwide are also descen-dants of these one or two females.

(D) It would depend on where they were discovered. If they were discovered in Hawaii, the Hawaiian fruitflies would still be considered the most taxonomically diverse population.

(E) There is insufficient information in the argument to allow you to determine the origins of fruit flies inother parts of the world. They may have originated in Hawaii, but you can’t demonstrate that conclu-sively from the argument.

rt LSATPrepTest 1

www.review.com 800-2-REVIEWwww.review.com 800-2-REVIEWCopyright © 1999 by Princeton Review Management, L.L.C. All Rights Reserved.

section 4question 3

Question 3: Identify the assumption. A feather is discovered in a limestone slab dating back to theMesozoic era. The author concludes that this is proof that birds existed during the Mesozoic era. This isan example of an interpretation of evidence question. The author assumes that there is no other connec-tion that can be drawn from the evidence (did the feather belong to some other kind of animal?).

(A) Although the author states that it was believed that birds appeared after the disappearance of thepterosaurs, nothing in the argument states that pterosaurs and birds are evolutionarily related.Instead, the pterosaurs are mentioned in the argument to help establish a timeline.

(B) Nothing is said about the species that may have preceded the animal from which the fossilized feathercame.

(C) This is the credited response. The only way for the author to claim that the feather came from a bird isfor the author to dismiss every other possible explanation.

(D) No, the author says the feather belonged to a bird. It would be pretty bizarre for the author to assumethat it didn’t belong to a bird in the face of that statement.

(E) No mention is made at all of how the bird might have flown, if it flew at all.

rt LSATPrepTest 1

www.review.com 800-2-REVIEWwww.review.com 800-2-REVIEWCopyright © 1999 by Princeton Review Management, L.L.C. All Rights Reserved.

section 4question 4

Question 4: Which of the following answer choices DOES NOT strengthen the argument? The answerchoice will either weaken the argument (this is what people generally expect to see) or it will be irrelevant tothe argument (this is what the test writers generally do). In this case, a decline in home energy consump-tion that followed an increase in oil prices is seen as evidence that people have reduced their standards ofliving or have changed the ways in which they spend their time. Four of the five answer choices will beexamples of either a reduced standard of living or a change in the ways in which people spend their time.

(A) This is an example of a change in the ways in which people spend their time. It’s also an example of areduced standard of living (people are no longer roaming freely within their own homes).

(B) This is an example of a change in the ways in which people spend their time.(C) This is the credited response. People have had to neither reduce their standards of living nor change

the ways in which they spend their time. Instead, the increase in the cost of oil was addressed byfamilies improving the efficiency of their heating systems.

(D) This is an example of a reduction in standard of living.(E) This is an example of a change in the way people spend their time.

rt LSATPrepTest 1

www.review.com 800-2-REVIEWwww.review.com 800-2-REVIEWCopyright © 1999 by Princeton Review Management, L.L.C. All Rights Reserved.

section 4question 5

Question 5: Describe Senator Strongwood’s reasoning. The senator cites a report done by his own partyto support his conclusion. He also states that the report he cites comes to a different conclusion than thestudy done by the administration. The senator concludes that the administration’s plan for reducing thecapital gains tax is dead on the strength of the report filed by Senator Strongwood’s party.

(A) Although the senator does claim that a reduction in the capital gains tax would increase the deficit, thesenator makes no claim about the effects of an increase in the capital gains tax.

(B) This is the credited response. The senator claims that other senators will follow the advice of hisparty’s report. If this happens, then senators have rejected the administration’s report.

(C) At no point does the senator expressly state that his opponents lack common sense. Instead hestates that the plan to reduce the capital gains tax is dead because presumably no senator lacks thecommon sense to approve the administration’s plan.

(D) This statement is too broad. The argument is about one piece of legislation and is not sufficientevidence to draw any kind of general claims.

(E) The senator’s only comparison between the two studies focuses on the accuracy, not the objectivityof the studies.

rt LSATPrepTest 1

www.review.com 800-2-REVIEWwww.review.com 800-2-REVIEWCopyright © 1999 by Princeton Review Management, L.L.C. All Rights Reserved.

section 4question 6

Question 6: Inference. Identify something that you know to be true from the argument.

(A) Since engineers are never mentioned in the argument, it’s not possible to draw any definitive conclu-sions about them.

(B) The fact that even Japan has a shortage of technically qualified people is not reason enough toconclude that Japan is not the best country against which to measure a country’s economic success.

(C) The fact that Japan’s base of highly skilled labor is insufficient to meet their needs does not mean thatthis base of labor is “uncommonly narrow”.

(D) This is the credited response. The beginning of the passage states that “the most successfuleconomies . . . will continue to be those that train as many people as possible in the human skillsrequired to research, to develop, and to apply new technology”.

(E) Most other countries are not mentioned in this argument, so it’s not possible to draw any definitiveconclusions about them.

rt LSATPrepTest 1

www.review.com 800-2-REVIEWwww.review.com 800-2-REVIEWCopyright © 1999 by Princeton Review Management, L.L.C. All Rights Reserved.

section 4question 7

Question 7: Identify an assumption. The argument’s main point is that a ceremonial game played with arubber ball in Mexico must have originated sometime after 1000 A.D. (when the pre-Columbian inhabitantsof Mexico began using rubber) but before 1519 A.D. (when Cortez observed inhabitants playing the game).The author of the argument assumes that the game was never played with anything but a rubber ball.

(A) Other ceremonial occasions are never discussed.(B) The argument centers around the earliest date the game could have been played. Whether this was

one of the earliest uses of rubber is not relevant to the argument.(C) No matter how popular the game was in Mexico, the argument only centers around the earliest date

the game could have been played. How widespread the game might have been is not relevant.(D) This is the credited response. The author of the argument dates the earliest possible inception of the

game by dating the earliest uses of rubber. If the game was played with a stone ball, for example, itcould be a much older game.

(E) There is no information given as to how we know that the earliest use of rubber in Mexico was aroundA.D. 1000.

rt LSATPrepTest 1

www.review.com 800-2-REVIEWwww.review.com 800-2-REVIEWCopyright © 1999 by Princeton Review Management, L.L.C. All Rights Reserved.

section 4question 8

Question 8: Which word are Jack and Mary using differently? The key to understanding this question isto note that Mary is from an outdoor advertising agency (the people who put up billboards), Jack is a localmerchant, and that they are attending a chamber of commerce meeting. The ambiguous word here is “our”.Mary uses “our” in reference to her specific business, while Jack uses “our” to refer to the businesses intown as a whole. The credited response is therefore (C).

rt LSATPrepTest 1

www.review.com 800-2-REVIEWwww.review.com 800-2-REVIEWCopyright © 1999 by Princeton Review Management, L.L.C. All Rights Reserved.

section 4question 9

Question 9: This kind of question has not appeared on the LSAT in about a hundred million years (okay,maybe that’s a bit of an exaggeration but it’s been a long time – long enough that you shouldn’t beparticularly concerned if you’re struggling with this question). But here’s an explanation anyway. Bycombining the last two statements (Anyone who is not a Montague is intemperate; no Montague isintemperate) you know that Montagues are not intemperate and everyone else is. Therefore, (E) is thecredited response because Capulets are not Montagues (this is the fourth statement in the argument). Insummary, if you are a Capulet you are not a Monatgue, and if you’re not a Montague then you are intem-perate.

rt LSATPrepTest 1

www.review.com 800-2-REVIEWwww.review.com 800-2-REVIEWCopyright © 1999 by Princeton Review Management, L.L.C. All Rights Reserved.

section 4question 10

Question 10: Parallel the flaw in the argument. The problem with the argument is that it takes two phenom-ena that occur together (in this case abundant rain in sub-Saharan Africa and hurricanes on the UnitedStates mainland) and presume a causal relationship when it is most likely that there is a third element that iscausing both observed phenomena (perhaps in this case it could be the high atmospheric winds off thewestern coast of Africa). Your credited response will propose a causal relationship that may be a case oftwo things being caused by a third (but unidentified) phenomenon. The closest parallel from among theanswer choices is answer choice (C). Playing competitive sports in college and becoming an entrepreneurmight be caused by a third factor (possibly a competitive spirit).

rt LSATPrepTest 1

www.review.com 800-2-REVIEWwww.review.com 800-2-REVIEWCopyright © 1999 by Princeton Review Management, L.L.C. All Rights Reserved.

section 4question 11

Question 11: Inference. Identify something you know to be true given what you are told in the passage.

(A) The fact that it is more important that we criticize democracies that commit human rights violationsthan that we criticize dictatorships that commit more serious offenses is not evidence that all govern-ments commit some inexcusable and reprehensible acts.

(B) This is the credited response. Less violent human rights offenses are more reprehensible if they arecommitted in a democracy than if they are committed in a dictatorship.

(C) Although our criticism is more likely to have an effect on a democracy, that should not be taken tomean that our criticism is certain to have no effect on a dictatorship.

(D) The relative likelihood of the occurrence of human rights violations in democracies versus dictator-ships is never discussed.

(E) We have no information about people who claim to represent the will of the people.

rt LSATPrepTest 1

www.review.com 800-2-REVIEWwww.review.com 800-2-REVIEWCopyright © 1999 by Princeton Review Management, L.L.C. All Rights Reserved.

section 4question 12

Question 12: Weaken the argument. The argument notes that smoking and snoring are more highlycorrelated than non-smoking and snoring. The author concludes that smoking by itself can cause snoring.The author assumes that two important things: first, it’s not the snoring that causes smoking; second,smoking and snoring aren’t being caused by some third thing. To weaken the argument, negate one of theassumptions.

(A) This is the credited response. It demonstrates that smoking and snoring are both effects of stress,thereby negating the second assumption cited above.

(B) But is obesity connected to snoring? The argument doesn’t say, so this can’t be the right answer.(C) This does not eliminate the possibility that for the snorers who do smoke that smoking has caused

them to snore.(D) Since the argument says that smoking can induce snoring, this statement does not weaken the

argument.(E) Throat problems are never mentioned in the argument.

rt LSATPrepTest 1

www.review.com 800-2-REVIEWwww.review.com 800-2-REVIEWCopyright © 1999 by Princeton Review Management, L.L.C. All Rights Reserved.

section 4question 13

Questions 13 and 14: Assumption (13) and Inference (14). The argument states that campaign advisersshould stay out of the limelight and let the press report on substantive issues the candidates have taken.Of course, for the press to report on these issues they must actually exist. Since this point is neverexplicitly made by the author of the argument, it can be classified as an assumption.

Question 13: Identify the assumption.

(A) Whether it is the most appropriate analogy is unimportant. The author does assume that chess is anaccurate enough analogy to illustrate the point, but the author could certainly accept that other(possibly better) analogies also exist.

(B) This is the credited response. If this statement were not true, the argument would make no sense.(C) The author’s point is not that press coverage determines the substantive issues of a campaign, it’s

that press coverage ignores the substantive issues of a campaign.(D) According to the author, voters are unable to make informed decisions because of the misapplied

focus of press coverage, not because of voters not paying enough attention to the election.(E) The author feels that there is a difference or there would be no argument.

rt LSATPrepTest 1

www.review.com 800-2-REVIEWwww.review.com 800-2-REVIEWCopyright © 1999 by Princeton Review Management, L.L.C. All Rights Reserved.

section 4question 14

Questions 13 and 14: Assumption (13) and Inference (14). The argument states that campaign advisersshould stay out of the limelight and let the press report on substantive issues the candidates have taken.Of course, for the press to report on these issues they must actually exist. Since this point is neverexplicitly made by the author of the argument, it can be classified as an assumption.

Question 14: Inference. Find something that you know to be true given the information in the argument.

(A) What candidates prefer is never discussed.(B) What the press enjoys is never discussed.(C) What candidates believe about political campaigning is never discussed.(D) What reporters find easier is never discussed.(E) This is the credited response. The author states that the current focus of election coverage is

“(depriving the voters) of the information they need to make informed decisions . . .”. Since thecurrent focus of election coverage is to focus on campaign advisors, you can draw this inference fromthe argument.

rt LSATPrepTest 1

www.review.com 800-2-REVIEWwww.review.com 800-2-REVIEWCopyright © 1999 by Princeton Review Management, L.L.C. All Rights Reserved.

section 4question 15

Question 15: Weaken the counterexample cited in the argument. The argument starts off by introducing aclaim that equal numbers of a given animal species are right-handed and left-handed. The author cites anapparent counterexample by citing the observation that dogs will almost always “shake hands” with theright paw. The author uses this observation to claim that dogs are predominantly right-handed. Toweaken this argument you’ll want to come up with another explanation for the observed phenomenon.

(A) Although this may support a claim that some dogs are ambidextrous, it doesn’t address thecounterexample cited in the argument.

(B) The issue here is not front versus rear paws, so this answer is not going to address thecounterexample very well.

(C) The inconvenience or stigma that dogs may or may not feel is not relevant to the argument.(D) How dogs compensate for the loss of a limb does not address the counterexample raised in the

argument.(E) This is the credited response. The reason that dogs appear to be right-handed when “shaking

hands” is that they were trained by right-handed people and the dog is just reacting to the behavior ofthe trainer who is extending her right hand.

rt LSATPrepTest 1

www.review.com 800-2-REVIEWwww.review.com 800-2-REVIEWCopyright © 1999 by Princeton Review Management, L.L.C. All Rights Reserved.

section 4question 16

Question 16: How has the student misinterpreted the professor’s remark? The professor states that if bothparents have type O blood, the offspring will have type O blood (both parents O ===> child O). Thestudent claims this statement cannot be true because he is type O and his father is type B. In effect, thestudent has heard the professor to say: child O ===> both parents O. Another way of stating thismisinterpretation is that if both parents are not type O then the child cannot be type O.

(A) This is the credited response. See the above discussion for more detail.(B) No children with type B blood are discussed.(C) If this is what the student thought the professor said then the student would have no objection.(D) Since nothing is said about other children in a family, this isn’t a good answer choice to pick.(E) This situation is never discussed in the argument, so it would be a bit of a stretch for the test writers

to expect you to discern this meaning from the student’s interpretation.

rt LSATPrepTest 1

www.review.com 800-2-REVIEWwww.review.com 800-2-REVIEWCopyright © 1999 by Princeton Review Management, L.L.C. All Rights Reserved.

section 4question 17

Question 17: Which answer does not resolve the paradox? R-Bar Beans are considered the best of allbrands and are preferred versus Texas T Beans and Aunt Sally’s Beans by a wide margin. Why is it thatTexas T and Aunt Sally’s sell much better nationally than R-Bar Beans do? One of the answer choices willnot provide an adequate explanation and that’s the answer you should pick.

(A) This helps explain why the national sales figures don’t reflect customer preference. People arechoosing their beans due to price, not brand preference.

(B) This is the credited response. Variance in preference for R-Bar Beans between age groups does notexplain why other brands are selling better.

(C) This helps explain why the national sales figures don’t reflect customer preference. R-Bar Beans arenot widely available, and national sales figures will reflect that reality.

(D) See (C).(E) This also helps explain why the national sales figures don’t reflect customer preference. R-Bar Beans

were only available the last three months of the year in which the sales figures were calculated, whilethe other brands were available for the entire year.

rt LSATPrepTest 1

www.review.com 800-2-REVIEWwww.review.com 800-2-REVIEWCopyright © 1999 by Princeton Review Management, L.L.C. All Rights Reserved.

section 4question 18

Question 18: Inference. Identify something you know to be true. Cosmetic firms have decided to shiftproduct testing away from using animal subjects and instead using cultures of human cells.

(A) Who initiated the pressure to cease animal testing doesn’t address the behavior described in theargument.

(B) This would make the behavior described in the argument appear to be a paradox. Why would thecosmetics companies make a change that would have no effect on sales?

(C) There’s nothing in the statement that would explain why the change is being considered.(D) See (C).(E) This is the credited response. This statement restates the idea that cosmetics firms are committed to

the active development of a testing technique that would allow them to cease animal testing withoutgoing beyond the argument to speculate as to why the change is being considered.

rt LSATPrepTest 1

www.review.com 800-2-REVIEWwww.review.com 800-2-REVIEWCopyright © 1999 by Princeton Review Management, L.L.C. All Rights Reserved.

section 4question 19

Questions 19 and 20: Main point (19) and describe the reasoning (20). The author states and then rejects aproposed relation between disliking the effects of aging and having a negative attitude towards the elderly.Rejecting the proposed relation is the main point of the argument.

Question 19: Main Point. The author feels there isn’t a necessary connection between disliking someconcomitants of the aging process and having a negative attitude towards the elderly.

(A) This is the credited response. See the above discussion for more detail.(B) This is a part of the argument used to support the main point.(C) Disliking the elderly because of their physical characteristics is never mentioned in the argument at all.

Instead, one of the supporting claims refers to disliking the elderly because they are old (which may ormay not be related to their physical characteristics – it’s not made clear).

(D) The argument has more to do with the perceptions of the elderly versus perceptions of some concomi-tants of the aging process.

(E) This is an element of the support of the main point of the argument.

rt LSATPrepTest 1

www.review.com 800-2-REVIEWwww.review.com 800-2-REVIEWCopyright © 1999 by Princeton Review Management, L.L.C. All Rights Reserved.

section 4question 20

Questions 19 and 20: Main point (19) and describe the reasoning (20). The author states and then rejects aproposed relation between disliking the effects of aging and having a negative attitude towards the elderly.Rejecting the proposed relation is the main point of the argument.

Question 20: The author does each of the following in the argument EXCEPT.

(A) The assertion that is dismissed is that people who don’t like the effects of aging have a more negativeattitude about old people.

(B) See the following quote from the argument: “Certainly it is reasonable to like the elderly yet dislike theidea of impaired eyesight and hearing.”

(C) See the following quote from the argument: “holding negative attitudes toward older people merelybecause they are old is immoral.”

(D) This is the credited response. It is questionable whether common stereotypes are ever mentioned,and if they are the author certainly does not discredit them.

(E) The author makes a distinction between attitudes towards the effects of aging and attitudes towardsthe elderly.

rt LSATPrepTest 1

www.review.com 800-2-REVIEWwww.review.com 800-2-REVIEWCopyright © 1999 by Princeton Review Management, L.L.C. All Rights Reserved.

section 4question 21

Question 21: Inference. Find something that must be true given the information in the passage.

(A) This isn’t necessarily true. It could just be that everyone is following the law.(B) See (A).(C) See (A).(D) This is the credited response. If a crime exists, then a law must exist (the one that was broken). In

arguments that use the word “some”, it is helpful to think of that word as meaning “at least one”.(E) This isn’t necessarily true. It could just be an instance of many violations of the same law.

rt LSATPrepTest 1

www.review.com 800-2-REVIEWwww.review.com 800-2-REVIEWCopyright © 1999 by Princeton Review Management, L.L.C. All Rights Reserved.

section 4question 22

Question 22: Identify a necessary assumption to the argument. The argument states that in droughtconditions that larger birds have a better chance of survival because they can crack open large seeds thatsmall birds cannot. Also, in periods of extended rain smaller birds have a better chance of survival becausethere is an abundance of small seeds that are too small to provide adequate sustenance to the large birds.For this hypothesis to be adequate, one must assume that there are not enough small seeds in droughtconditions to support the small birds and not enough large seeds in rainy conditions to support the largebirds.

(A) Small, hard seeds would wipe out all the birds: the seeds are too hard to be useful to small birds andtoo small to be useful to large birds.

(B) This is the credited response. See the above analysis for more details.(C) This answer choice does nothing to explain why the larger birds don’t thrive during rainy periods.(D) But why do the climatic changes affect the survival rates of large birds during rainy periods?(E) See (D).

rt LSATPrepTest 1

www.review.com 800-2-REVIEWwww.review.com 800-2-REVIEWCopyright © 1999 by Princeton Review Management, L.L.C. All Rights Reserved.

section 4question 23

Question 23: Strengthen Ms. Fring’s argument. Mr. Blatt claims that consultants command substantialfees because their advice is so valuable. Ms. Fring counters this by claiming that executives hire expen-sive consultants so they can blame the consultants if something goes wrong. Note that the difference ofopinion between the two centers on why executives pay substantial fees to consultants. You are lookingfor an answer that supports the connections between high fees and “blameworthiness”.

(A) Do they hire the consultant for the quality of advice, or are they acquiring a scapegoat in casesomething goes wrong? This example does nothing to resolve the argument.

(B) This doesn’t explain why the consultant was (or wasn’t) hired.(C) This is the credited response. If managers are hiring consultants based on the quality of the advice

they give, then the firm should have seen at least as much business as they did before they cut theirprices. The fact that business declined in the face of a discounted price supports the contention thatmanagers are attracted to high-priced consultants so they can blame someone else if things go wrong(the cheaper consultants aren’t as “blameworthy”).

(D) If anything, this supports Mr. Blatt’s argument since it values the consultant based on the quality ofthe advice the consultant gives.

(E) See (A).

rt LSATPrepTest 1

www.review.com 800-2-REVIEWwww.review.com 800-2-REVIEWCopyright © 1999 by Princeton Review Management, L.L.C. All Rights Reserved.

section 4question 24

Question 24: Parallel the reasoning. In formal terms, the original argument is as follows – All A are B.Some A are C. Therefore, some B are C. The parallel argument, although the statements are presented inthe reverse order, is answer choice (E). In answer choice (E), the statements are presented in the followingorder – (Therefore) Some B are C. (because) Some A are C. (and) All A are B.